You are on page 1of 201

National Board of Medical Examiners^

Comprehensive Basic Science Self-Assessment

NATIONAL BOARD OF MEDICAL EXAMINERS


TERMS, CONDITIONS AND DISCLAIMERS

for the

Comprehensive Basic Science Self-Assessment


You must read this section before proceeding . Please read this agreement carefully before proceeding. By participating in the self-assessment you attest that you agree to be bound by the terms and conditions below.

The material presented in the Comprehensive Basic Science Self-Assessment is provided by National Board of Medical Examiners (NBME®) for educational purposes only. The multiple choice questions presented on the assessment are based on information
typically covered during basic science medical education and are intended for individuals who have participated in these educational experiences. This self-assessment resembles the content of USMLE® Step 1, and medical students may find this to be a valuable
tool in structuring their study as they prepare to take USMLE Step 1 . The self-assessment is not intended to predict performance on USMLE. Rather the self-assessment is intended to be used as a tool to determine a participant’s relative areas of strengths and
weaknesses in general topic areas.

Under no circumstances shall NBME be liable for any damages or costs that may result in any way from your reliance on or use of the information derived from your participation in the self- assessment service .

This examination contains test materials that are owned and copyrighted by NBME . Any reproduction of these materials or any part of them through any means: including but not limited to . copying or printing of electronic files, reconstruction through memorization
and/or dictation . and/or dissemination of these materials or any part of them . is strictly prohibited .

NBME reserves the right to pursue its rights and remedies to the fullest extent permitted by law without further notice to you, including taking legal action.

By clicking "Yes” on the button below and by participating in the self-assessment you acknowledge that you have read understood and agreed to be bound by these Terms, Conditions and Disclaimers . If you do not wish to be bound by these Terms: Conditions and
;

Disclaimers, you should not participate in the assessment .

No
o
Yes Help Pause
Exam Section 1: Item 1 of 50 National Board of Medical Examiners^ Time Remaining:
Mark Comprehensive Basic Science Self-Assessment 1 hr 14 min 58 sec

1 . A 75 -year-old man has decreased exercise tolerance and occasional syncopal episodes related to exertion . Left-heart catheterization discloses this pressure tracing (Ao -aorta : LV - left ventricle ; LA -
left atrium) . The most frequently encountered mechanism underlying this disorder is which of the following?

0 A ) Amyloid deposition 150 -ri


t
6 } Calcification
C C ) Cystic medial necrosis
u>
D ) Pericardial effusion
e 100 -
E } Septal hypertrophy 1
e
<D
*
I
Ao
i
i
I
>
(/
</> 50 - i
I
LV
0)
* LA
i
tx J
* * **
* *** r * **** * *** * »\
*
0J
Time

Next Lab Values Calculator Review Help Pause


Exam Section 1: Item 2 of 50 National Board of Medical Examiners^ Time Remaining:
Mark Comprehensive Basic Science Self-Assessment 1 hr 14 min 54 sec

2 . New reagents are being designed to treat graft-versus -host disease in HLA - unrelated bone marrow transplant recipients. Reagents should be designed that selectively deplete which of the following donor cells ?

0 A ) B lymphocytes
B ) Dendritic cells
0 C ) Macrophages
D ) Neutrophils
E ) T lymphocytes

o
Previous
o
Next Lab Values
I!
Calculator
&
Review Help Pause
Exam Section 1: Item 3 of 50 National Board of Medical Examiners ^ Time Remaining:
Mark Comprehensive Basic Science Self-Assessment 1 hr 14 min 52 sec

3 . A 9-year-old boy has had alopecia and hypocalcemia since birth . His serum 1 25- dihydroxycholecalciferol concentration is within the reference range: serum 24 25- dihydroxycholecalciferol is undetectable. Serum parathyroid hormone concentration is above
; ;

the upper limit of the laboratory assay and needs to be reassayed at a dilution . The results are pending . Which of the following is the most likely cause of his condition?

O A ) Loss-of -function mutation in the calcium-sensing receptor


B ) Multiple endocrine neoplasia type I
O C ) Mutations inactivating the vitamin D 24-hydroxylase gene
! _ D ) Mutations inactivating the vitamin D receptor

o
Previous
o
Next
E
(
Lab Values
I!
Calculator
A
Review Help Pause
Exam Section 1: Item 4 of 50 National Board of Medical Examiners^ Time Remaining:
Comprehensive Basic Science Self-Assessment 1 hr 14 min 50 sec

4 . Assuming the positive predictive value of a test is 50% and the negative predictive value is 75 % which of the following tables are consistent with these values ?
;

Disease Disease
Present Absent P resent Absent

A B
Positive 50 50 750 250
Test
Negative 250 750 50 50

C D
Positive 250 750 50 50
Test
Negative 50 50 750 250

E Cannot be determined from data given

O A)
O B)
O C)
O D)
O E)

o
Previous
o
Next Lab Values
*
Calculator Review Help Pause
Exam Section 1: Item 5 of 50 National Board of Medical Examiners^ Time Remaining:
Mark Comprehensive Basic Science Self-Assessment 1 hr 14 min 47 sec

5 . A 43-year-old woman comes to the physician because of vague discomfort, nausea, easy bruising and weight loss . Her prothrombin time is increased . The most likely cause of the increased prothrombin time is damage to which of the following cells ?
;

0 A ) Endothelial cells
B ) Fibroblasts
0 C ) Hepatocytes
D ) Macrophages
E ) Skeletal muscle cells
F ) T lymphocytes

o
Previous
o
Next
(E
Lab Values
I!
Calculator
A
Review Help Pause
Exam Section 1: Item 6 of 50 National Board of Medical Examiners ^ Time Remaining:
Mark Comprehensive Basic Science Self-Assessment 1 hr 14 min 45 sec

6 . A 63-year-old man has had this gradually enlarging lesion on his right forearm for the past 3 years. Which of the following is the most likely diagnosis ?

C A ) Actinic keratosis
6 } Blue nevus
(_) C ) Compound nevus
D ) Dermatofibroma
E ) Halo nevus
F ) Hemangioma
C G ) Malignant melanoma
H ) Pyogenic granuloma
C I ) Seborrheic keratosis
' '

_ J ) Squiamous cell carcinoma

Previous
o o
Next Lab Values
A
Calculator
A
Review
r
Help * r*
Pause
Exam Section 1: Item 7 of 50 National Board of Medical Examiners ^ Time Remaining:
Mark Comprehensive Basic Science Self-Assessment 1 hr 14 min 42 sec

7 . A 6G -year-old man has a surgical excision of a brain mass ; a photomicrograph of excised tissue is shown He has smoked 1 pack of cigarettes daily for the past 50 years. His blood pressure is 130/ 90 mm Hg .
Laboratory studies show polycythemia, hypercalcemia , and microscopic hematuria. Immunostaining of the tissue is positive for epithelial membrane antigen and negative for carcinoembryonic antigen . Which of the
following is the most likely site of the primary neoplasm ?

G A ) Colon
6 } Kidney
O C ) Liver
D ) Lung
E ) Lymph node
C F ) Prostate
O G ) Testis

'

"• *
j

o
Previous
o
Next Lab Values Calculator
m
Review Help Pause
Exam Section 1: Item 8 of 50 National Board of Medical Examiners ^ Time Remaining:
Mark Comprehensive Basic Science Self-Assessment 1 hr 14 min 40 sec

8 . In a study designed to test the effectiveness of a new drug in the treatment of endometriosis, 100 women are randomly assigned to one of two groups : 48 of the women receive the new drug and 52 receive standard therapy. Which of the following is the major
benefit of assigning patients in this manner?

O A ) The effect of confounding variables is decreased


B ) The likelihood of achieving statistical significance is increased
O C ) The power of the study is increased
! _ D ) The study groups are equalized

o
Previous
o
Next Lab Values
I!
Calculator
&
Review Help Pause
Exam Section 1: Item 9 of 50 National Board of Medical Examiners ^ Time Remaining:
Mark Comprehensive Basic Science Self-Assessment 1 hr 14 min 37 sec

9 . A 17-year-old girl who is a gymnast is brought to the emergency department 30 minutes after fainting during a competition . Her mother says that the patient eats so little food that her teeth are decaying and that she has stopped menstruating . She is 165 cm
(5 ft 5 in) tall and weighs 37 kg (82 lb) ; BMI is 14 kg/ m 2 . Sexual development is Tanner stage 2. Her pulse is 130/ min and blood pressure is 80/50 mm Hg . Physical examination shows pale, cool skin . There are scars across her knuckles that she says are cat
scratches. Serum studies show a potassium concentration of 2.9 mEq/ L and a chloride concentration of 92 mEq /L. Which of the following is the most likely diagnosis ?

O A ) Anorexia nervosa (binge eating/ purging type)


o B ) Anorexia nervosa (restricting type)
o C ) Bulimia nervosa (nonpurging type)
o D ) Chronically high metabolic demands of competition with dehydration during the meet
o E ) Obsessive compulsive disorder with food compulsions
-

o
Previous
o
Next
(£5
Lab Values
e
Calculator
©
Review
&
Help Pause
Exam Section 1: Item 10 of 50 National Board of Medical Examiners ^ Time Remaining:
Mark Comprehensive Basic Science Self-Assessment 1 hr 14 min 34 sec

10. A fetus exposed to an infection at 22 weeks ' gestation appears normal at birth but later develops a persistent mucopurulent nasal discharge malformed teeth skin cracks and fissures around oral, anal , and vulvar orifices, splenomegaly and
; ,
;

lymphadenopafhy. Which of the following best explains these findings ?

O A ) Inability of maternal rubella antibodies to cross the placental barrier


B ) Increased amounts of herpes simplex virus in amniotic fluid
O C ) Maternal failure to generate antibodies to cytomegalovirus
D ) Maternally transferred spirochetes replicating in fetal tissue
! E ) Persistence of HIV in placental tissue

o
Previous
o
Next
E
(
Lab Values
I!
Calculator
A
Review
r
Help
&
Pause
Exam Section 1: Item 11 of 50 National Board of Medical Examiners^ Time Remaining:
Mark Comprehensive Basic Science Self-Assessment 1 hr 14 min 32 sec

11 . The biological effects of thyroid hormone and retinoic acid are mediated through similar mechanisms. Which of the following properties is common to the receptors for these two compounds ?

0 A ) Ability to activate phospholipase C


B ) DNA - binding domain
0 C ) Formation of a dimer on binding of hormone
D ) Interaction with a G protein
0 E ) Tyrosine kinase activity

o
Previous
o
Next
E
(
Lab Values Calculator
A
Review
r
Help
&
Pause
Exam Section 1: Item 12 of 50 National Board of Medical Examiners ^ Time Remaining:
Mark Comprehensive Basic Science Self-Assessment 1 hr 14 min 29 sec

12 . A 57 -year-old man conies to the physician because of an intermittent cough and a 9-kg (20- lb ) weight loss over the past 6 months . He has smoked 1 pack of cigarettes daily for 42 years. His serum calcium concentration is 13.3 mg/dl_ . An x- ray of the chest
shows a 4-cm central mass in the left lung . Which of the following is the most likely diagnosis ?

O A ) Adenocarcinoma of the lung


B ) Large cell lymphoma
O C ) Metastatic osteosarcoma
D ) Metastatic renal cell carcinoma
E ) Small cell carcinoma of the lung
;_! F ) Squamous cell carcinoma of the lung

o
Previous
o
Next
E
(
Lab Values
I!
Calculator
A
Review Help Pause
Exam Section 1: Item 13 of 50 National Board of Medical Examiners ^ Time Remaining:
Mark Comprehensive Basic Science Self-Assessment 1 hr 14 min 27 sec

13. A 45 -year-old woman has had progressive fatigue and mild shortness of breath with exertion during the past 3 months . She thinks she had rheumatic fever at the age of 10 years . A grade 2/6 : rumbling diastolic murmur is heard at the fifth left intercostal
;

space in the midclavicular line . A lateral x-ray of the chest shows mild posterior displacement of the esophagus . Which of the following cardiovascular structures is most likely responsible for the esophageal displacement?

O A ) Descending aorta
B ) Left atrium
C ) Left ventricle
D ) Right atrium
E ) Right ventricle

o
Previous
o
Next Lab Values Calculator
m
Review
c*
Help
it*
Pause
Exam Section 1: Item 14 of 50 National Board of Medical Examiners ^ Time Remaining:
Mark Comprehensive Basic Science Self-Assessment 1 hr 14 min 24 sec

14 . A 34-year-old woman is brought to the emergency department by her husband because of ain inability to see objects in the peripheral visual fields in both eyes for 1 day. Her temperature is 37.2 aC (99°F ), pulse is G 8/min . respirations are 16/min , and blood
pressure is 124,78 mm Hg. Automated computerized perimetry examination shows a loss of the temporal visual field in both eyes. Neurologic examination shows no other abnormalities . This patient is most likely to have which of the following additional
findings?

A ) Amenorrhea
B ) Aphasia
C ) Double vision
D ) Loss of memory
E ) Seizures

o
Previous
o
Next Lab Values Calculator
m
Review
C*
Help
it*
Pause
Exam Section 1: Item 15 of 50 National Board of Medical Examiners ^ Time Remaining:
Mark Comprehensive Basic Science Self-Assessment 1 hr 14 min 22 sec

15. A 46 -year-old man with type 2 diabetes mellitus is brought to the emergency department by his wife 30 minutes after he developed chest pain and then lost consciousness . He has a 2- year history of exercise-induced angina pectoris that is well controlled with
isosorbide dinitrate: atenolol and diltiazem. Current medications also include metformin and glipizide . His wife is tearful and says, "He took one of his friend' s Viagra pills about 4 hours ago." On arrival , he is alert . His blood pressure is 60/30 mm Hg . Physical
examination shows pallor and diaphoresis. This patient is most likely having which of the following effects of a drug interaction?

A ) Bradycardia
B ) Paroxysmal supraventricular tachycardia
C ) Third-degree atrioventricular block
D ) Torsades de poirtes
E ) Vasodilation

o
Previous
o
Next
It?
Lab Values
*
Calculator
o
Review Help Pause
Exam Section 1: Item 16 of 50 National Board of Medical Examiners ^ Time Remaining:
Mark Comprehensive Basic Science Self-Assessment 1 hr 14 min 20 sec

16. A 22 -year-old woman is admitted to the hospital because of a 10- day history of polydipsia and polyuria . She says that the urge to urinate often awakens her at night . She has been taking lithium carbonate for 2 years for bipolar disorder : her dosage was
increased 6 months ago because of recurrent severe manic episodes . Her vital signs are within normal limits. Physical examination shows no abnormalities . Over the next 24 hours: urine excretion totals 6.5 L . Laboratory studies at this time show a serum
sodium concentration of 148 mEq /IL . serum osmolality of 315 mOsmol/kg , and urine osmolality of 75 mOsmol/kg . After administration of desmopressin, urine output and osmolality do not change. Which of the following mechanisms is the most likely cause of
the polyuria in this patient?

C) A ) Decreased secretion of ADH (vasopressin)


o B ) Inhibition of cAMP mediated processes iin the collecting duct
-

o C ) Osmotic diuresis due to increased amounts of glucose in the tubule


o D ) Polydipsia induced washout of the renal interstitial concentration gradient
-

o E ) Renal medullary necrosis due to decreased papillary blood flow

o
Previous
o
Next
It?
Lab Values
*
Calculator
®
Review
r
Help
r
Pause
Exam Section 1: Item 17 of 50 National Board of Medical Examiners ^ Time Remaining:
Mark Comprehensive Basic Science Self-Assessment 1 hr 14 min 18 sec

17. A clinical study compares patients with HIV infection with an age-matched control group of persons without HIV infection. Men and women in both the study and control groups have a history of unprotected sexual intercourse with both male and female
partners. The CD 4+ T-lymphocyte courts of persons in both groups are greater than 50 G/ mm 3 (N>500) The histories of any infection are obtained from persons in both groups. A history of infection with which of the following organisms most likely increased
the risk for developing HIV infection in the study group?

O A ) Burkhoideria cepacia
B ) Candida albicans
O C ) Cryptococcus neoformans
D ) Cytomegalovirus
E } Haemophilus ducreyi
'

_ F ) Trichophyton rubrum

o
Previous
o
Next Lab Values Calculator
m
Review Help Pause
Exam Section 1: Item 13 of 50 National Board of Medical Examiners^ Time Remaining:
Mark Comprehensive Basic Science Self-Assessment 1 hr 14 min 15 sec

18. A 77-year-old woman dies in the hospital after a long illness. Her vertebral column , obtained at autopsy is shown in the photograph. The process shown is most likely associated with an increase in which of the
following?

A ) Calcium
6 } Estrogen
C C ) lnterleukin-1 (IL- 1)
D ) Monoclonal immunoglobulin
E ) Vitamin D

m
Previous Next Lab Values Calculator Review
*
Help *T*
Pause
Exam Section 1: Item 19 of 50 National Board of Medical Examiners ^ Time Remaining:
Mark Comprehensive Basic Science Self-Assessment 1 hr 14 min 12 sec

19. A IQ -year-old boy is brought to the physician because of a 3- day history of difficulty walking because his right foot drops when he lifts it . He is a member of a wrestling team at his high school . Physical examination shows weakness of the right ankle
dorsiflexor muscles . The right ankle evertor muscles have full power. Sensation to pinprick is decreased between the great and second toes of the right foot . Sensation over the rest of the foot is normal . Which of the following nerves is most likely damaged in
this patient?

A ) Common fibular ( peroneal )


B ) Deep fibular (peroneal)
C ) Sciatic
D ) Superficial fibular (peroneal)
E ) Tibial

o
Previous
o
Next
E
Lab Values Calculator
m
Review
r
Help *r*
Pause
Exam Section 1: Item 20 of 50 National Board of Medical Examiners ^ Time Remaining:
Mark Comprehensive Basic Science Self-Assessment 1 hr 14 min 10 sec

20. An 35 - year - old man comes to the physician because of a 4-month history of headaches shortness of breath and leg swelling . He has a 15 -year history of well-controlled hypertension. His pulse is 30/min . respirations are 18/min, and blood pressure is
; :

210/110 mm Hg . Crackles are heard at the lung bases . There is a bruit in the left flank and edema in both lower extremities. Urinalysis shows :
Specific gravity 1.015 (N=1 001-1 038 )
Blood trace
Protein 30 mg/ 24 h
RBC 6/hpf
WBC Q/hpf
Bacteria negative

He dies the next day. Which of the following findings is most likely on autopsy ?

£ ) A ) Chronic interstitial nephritis


o B ) Ostial stenosis of one renal artery
o C ) Scattered nodular aneurysms in the renal arteries
D ) Solitary left kidney

o
Previous
o
Next
It?
Lab Values
*
Calculator
®
Review
r
Help
r
Pause
Exam Section 1: Item 21 of 50 National Board of Medical Examiners^ Time Remaining:
Mark Comprehensive Basic Science Self-Assessment 1 hr 14 min 7 sec

21 . This graph best depicts the natural history of which of the following human disorders?

C A ) Creutzfeldt-Jakob disease
E
6 } Guillain - Barre syndrome o
Q_
O C ) HIV infection E
D ) Subacute sclerosing panencephalitis <n

E ) Yellow fever o

Time
—— ClintMl disease (tireshold
Course of disease
Actiyfl production of infectious agem

o
Previous
o
Next Lab Values
!
I
Calculator
A
Review Help Pause
Exam Section 1: Item 22 of 50 National Board of Medical Examiners ^ Time Remaining:
Mark Comprehensive Basic Science Self-Assessment 1 hr 14 min 5 sec

22 . A 63-year -old woman comes to the physician because of vaginal bleeding for 2 weeks. Menopause occurred at the age of 46 years , and she received hormone replacement therapy for 4 years. She has osteoarthritis of the hands and knees: but she has been
otherwise healthy. Pelvic examination shows a mildly enlarged uterus and a 1 G- cm right adnexal mass . An endometrial biopsy specimen shows simple hyperp asia without atypia . This patient most likely has which of the following types of adnexal masses ?

O A ) Dysgerminoma
B ) Granulosa cell tumor
O C } Mature cystic terato m a
D ) Serous cystadenocarcinoma
' _ ! E } Sertoli - Leydig cell tumor

o
Previous
o
Next
E
(
Lab Values
I!
Calculator
A
Review Help Pause
Exam Section 1: Item 23 of 50 National Board of Medical Examiners ^ Time Remaining:
Mark Comprehensive Basic Science Self-Assessment 1 hr 14 min 2 sec

23. During a study on exercise: a 45- year -old woman ruins on a treadmill for 30 minutes. An increase in which of the following is most likely to change gastrocnemius muscle blood flow in this woman?

0 A ) Interstitial adenosine concentration


B ) Interstitial amino acid concentration
0 C ) I nte rsti ti al g Iucose concentration
D ) Interstitial oxygen tension
E ) Parasympathetic stimulation
F ) Sympathetic stimulation

o
Previous
o
Next
E
(
Lab Values
I!
Calculator
A
Review
r
Help
&
Pause
Exam Section 1: Item 24 of 50 National Board of Medical Examiners ^ Time Remaining:
Mark Comprehensive Basic Science Self-Assessment 1 hr 14 min 0 sec

24 . A 52 -year -old woman with breast cancer comes to the physician for a follow- up examination . She had a 4-week course of radiation treatment 6 months ago. Her respirations are 26,'min . Physical examination shows no recurrence of the cancer. A CT scan of
the chest shows bilateral patches of atelectasis in the upper lung fields . The atelectasis in this patient most likely developed because of which of the following primary pathophysiologic processes?

A ) Compression
B ) Consolidation
C ) Contraction
D ) Obstruction
E ) Resorption

o
Previous
o
Next Lab Values Calculator
A
Review
C*
Help
it*
Pause
Exam Section 1: Item 25 of 50 National Board of Medical Examiners ^ Time Remaining:
Mark Comprehensive Basic Science Self-Assessment 1 hr 13 min 53 sec

25. A 50 -year -old woman comes to the physician with her husband because of a G -month history of progressive weakness of her arms and legs. Neurologic examination shows mild spasticity weakness: and hyperreflexia of the upper and lower extremities. A
;

Babinski sign is present bilaterally. An MR! of the spine shows a 1 5- cm , well - defined spherical lesion indenting the cervical spinal cord on the left at the C 1 level. Which of the following is the most likely diagnosis ?
,

O A ) Arteriovenous malformation
B ) Astrocytoma
O C ) Epidural abscess
D ) Herniated disc
E ) Meningioma

o
Previous
o
Next Lab Values Calculator
m
Review Help Pause
Exam Section 1: Item 26 of 50 National Board of Medical Examiners ^ Time Remaining:
Mark Comprehensive Basic Science Self-Assessment 1 hr 13 min 56 sec

26. A 33 -year -old woman who is undergoing intensive chemotherapy for acute myelogenous leukemia develops pancytopenia . She also has pain tingling, and itching of the skin of the right lower lip. One day later a group of vesicles containing clear fluid appears
:

on the lower lip. The most appropriate pharmacotherapy for this patient is a drug that inhibits which of the following?

A ) DNApolymerase
B ) Neuraminidase
C ) Protease
D ) Pyrophosphatase
' _ ! E } Reverse transcriptase

o
Previous
o
Next Lab Values Calculator
m
Review
c*
Help
it*
Pause
Exam Section 1: Item 27 of 50 National Board of Medical Examiners ^ Time Remaining:
Mark Comprehensive Basic Science Self-Assessment 1 hr 13 min 54 sec

27. A 24-year -old man conies to the physician because of fever muscle aches, runny nose. cough . and chills for 1 day. His temperature is 39. SX (103.8T ), pulse is 92 /min respirations are 24/ min and blood pressure is 124/78 mm Hg . Physical examination
; :

shows erythema of the throat but no pus. The lungs are clear to auscultation . Which of the following is the most likely cause of the chills in this patient?

O A ) An abrupt alteration in the cellular components of the blood


B ) An abrupt alteration in the oxygenation of the blood
O C ) An abrupt alteration of the hypothalamic thermoregulatory center
D ) Viral infection of the brain
E ) Viral infection of the muscles
F ) Viral infection of the vascular endothelial cells

o
Previous
o
Next
(E
Lab Values
I!
Calculator
A
Review Help Pause
Exam Section 1: Item 23 of 50 National Board of Medical Examiners ^ Time Remaining:
Mark Comprehensive Basic Science Self-Assessment 1 hr 13 min 52 sec

28. A 23 -year -old woman comes to the physician because of a 2 -week history of generalized swelling . She has autoimmune (Hashimoto) thyroiditis and chronic hepatitis C. Her temperature is 37.1 DC (98.7°F), pulse is 74/min and blood pressure is
;

142/33 mm Hg . Physical examination shows 2 + peripheral pitting edema . Laboratory studies show a serum albumin concentration of 2.1 g/dL: serum cholesterol concentration of 324 mg/dL , and 3+ protein in urine. A renal biopsy specimen shows diffuse
thickening of the glomerular capillary wall, granular immunofluorescent deposits of IgG along the glomerular basement membrane , and irregular subepithelial electron- dense deposits . This patient most likely has which of the following types of
glomerulonephritis ?

C) A ) Crescentic
o B ) Membranous
o C ) Mesangioproliferative
o D ) Proliferative
o E ) Rapidly progressive

o
Previous
o
Next
It?
Lab Values
*
Calculator
®
Review
f
Help Pause
Exam Section 1: Item 29 of 50 National Board of Medical Examiners ^ Time Remaining:
Mark Comprehensive Basic Science Self-Assessment 1 hr 13 min 49 sec

29. A 67 -year -old man is brought to the physician because of a 2-day history of double vision and drooping of his left eyelid. He has no history of trauma to the eye. His vital signs are within normal limits. Physical examination shows ptosis of the left eyelid . When
the eyelid is raised manually, the eye is fixed in the out position and the pupil is dilated. The visual acuity of the left eye is within normal limits . Which of the following is the most likely cause of these findings ?

O A ) Aneurysm of the posterior communicating artery


B ) Compression of the superior cervical ganglion
O C ) Damage to the trochlear nerve
D ) Occlusion of the scleral venous sinus
E ) Tumor of the optic nerve

o
Previous
o
Next Lab Values Calculator
m
Review
c*
Help
it*
Pause
Exam Section 1: Item 30 of 50 National Board of Medical Examiners ^ Time Remaining:
Mark Comprehensive Basic Science Self-Assessment 1 hr 13 min 46 sec

30. A 2 S -year -old man conies to the physician because of a 3-week history of numbness of his penis and difficulty achieving and maintaining an erection . He is a competitive bicyclist who rides at least 20 to 30 miles daily. Physical examination shows no
abnormalities . This patient's symptoms are most likely caused by increased pressure on the branches of which of the following nerves ?

A ) Genitofemoral
B ) Ilioinguinal
C ) Inferior gluteal
D ) Pudendal
E ) Sacral splanchnic

o
Previous
o
Next Lab Values Calculator
m
Review
c*
Help
it*
Pause
Exam Section 1: Item 31 of 50 National Board of Medical Examiners ^ Time Remaining:
Mark Comprehensive Basic Science Self-Assessment 1 hr 13 min 44 sec

31. A 62 -year -old man conies to the physician for a follow -up examination . Two weeks ago he was discharged from the hospital after sustaining an acute myocardial infarction. He smokes 2 packs of cigarettes and drinks four 12 -oz beers daily. His diet mostly
;

consists of cured meats and fast food . He does not exercise . The patient tells the physician . " I know that I need to make some changes in how I live so that my heart can be healthier. I just don't have the willpower to quit smoking and drinking and all that stuff
right now. " Which of the following best describes this patient’s stage of behavioral change ?

A ) Precontemplation
B ) Contemplation
C ) Preparation
D ) Action
E ) Maintenance

o
Previous
o
Next Lab Values Calculator
m
Review
c*
Help
it*
Pause
Exam Section 1: Item 32 of 50 National Board of Medical Examiners ^ Time Remaining:
Mark Comprehensive Basic Science Self-Assessment 1 hr 13 min 42 sec

32 . A 35 -year -old man conies to the physician because of a 2 -year history of slowly progressive muscle weakness of hiis thighs . His father and paternal uncle have a disorder involving proximal muscle weakness of the lower extremities. Physical examination
shows no other abnormalities . A muscle biopsy specimen shows ragged red fibers .. Electron- microscopic examination of this specimen is most likely to show which of the following inclusions ?

O A ) Intranuclear filamentous
B ) Lysosomal
O C ) Mitochondrial crystalline
D ) Paranuclear filamentous
E ) Sarcoplasmic reticular

o
Previous
o
Next
(E
Lab Values
!
I
Calculator
A
Review
r
Help
&
Pause
Exam Section 1: Item 33 of 50 National Board of Medical Examiners ^ Time Remaining:
Mark Comprehensive Basic Science Self-Assessment 1 hr 13 min 39 sec

33. A 32 -year -old man is being evaluated for increased jugular venous pressure: a systolic murmur hepatomegaly, ascites, and 2+ pitting edema of the ankles. He is an intravenous drug user. Which of the following is the most likely diagnosis ?
,

A ) Aortic regurgitation
B ) Aortic stenosis
C ) Chronic anemia
D ) Mitral regurgitation
E ) Mitral stenosis
F ) Tricuspid regurgitation

o
Previous
o
Next
(E
Lab Values
!
I
Calculator
A
Review
&
Help
tl*
Pause
Exam Section 1: Item 34 of 50 National Board of Medical Examiners ^ Time Remaining:
Mark Comprehensive Basic Science Self-Assessment 1 hr 13 min 36 sec

34 . A 3G -year -old woman , gravida 2 , para 1 who is Rh-negative delivers a male stillborn at 24 weeks’ gestation . She had no prenatal care. The stillborn is Rh-positive. Autopsy findings are consistent with erythroblastosis fetalis . The mechanism of this reaction is
;

most similar to that found in which of the following conditions ?

O A ) Graft-versus- hostdisease
B ) Graves disease
C ) Peanut allergy
D ) Poison ivy dermatitis
E ) Tuberculin reaction

o
Previous
o
Next
E
(
Lab Values
I!
Calculator
A
Review Help Pause
Exam Section 1: Item 35 of 50 National Board of Medical Examiners^ Time Remaining:
Mark Comprehensive Basic Science Self-Assessment 1 hr 13 min 34 sec

35. A 1-day-old male newborn has bilious vomiting and reluctance to feed . Physical examination shows dehydration and mild abdominal distention . Abdominal x-rays show a volvulus of the small bowel between the third and fourth segments of the duodenum.
Which of the following blood vessels is most likely constricted by the volvulus in this patient?

O A ) Celiac trunk
B ) Inferior mesenteric artery
O C ) Inferior vena cava
D ) Splenic vein
' _ ! E } Superior mesenteric artery

o
Previous
o
Next Lab Values Calculator
m
Review Help Pause
Exam Section 1: Item 36 of 50 National Board of Medical Examiners ^ Time Remaining:
Mark Comprehensive Basic Science Self-Assessment 1 hr 13 min 32 sec

36. A 79-year -old woman with osteoarthritis comes to the physician for an initial examination . She is otherwise healthy and lives alone . When the physician enters the examination room he detects a siight odor suggestive of urine. It is most appropriate for the
,

physician to approach the topic of urinary incontinence with this patient in which of the following ways?

O A ) "Are you having any problems that you may be too embarrassed to talk about? Would you be willing to talk about them today?"
o B ) 1 couldn't help but notice an odor when I came into the room Are you having any difficulty holding your urine?"
' .

o C ) "I understand this can be difficult to talk about but do you find yourself also having fecal incontinence in addition to urinary incontinence?"
o D ) "I know this can be an embarrassing topic, but people sometimes have difficulty holding their urine as they get older Is this ever a problem for you?"
.

o E ) "It is not surprising that someone your age is having problems with urinary incontinence How many accidents do you have a day ?
. .
11

o
Previous
o
Next Lab Values
*
Calculator
©
Review
&
Help Pause
Exam Section 1: Item 37 of 50 National Board of Medical Examiners ^ Time Remaining:
Mark Comprehensive Basic Science Self-Assessment 1 hr 13 min 30 sec

37. A 49-year -old woman comes to the emergency department because of a 3-day history of fever shortness of breath , and confusion . She is a postal worker. Her temperature is 38.4X (101 2°F): respirations are 28/min , and blood pressure is 100/60 mm Hg .
Physical examination shows nuchal rigidity. Breath sounds are decreased on the right side of the clhest. A lumbar puncture is done. Analysis of cerebrospinal fluid ( CSF) shows:
Glucose 18 mg/d L
Total protein 138 mg/dL
Leukocyte count 638/mm 3
Segmented neutrophils 87%
Monocytes 13%
RBC 2300/mm 3

A Gram stain of CSF shows large, gram -positive spore- forming bacilli . A chest x-ray shows marked widening of the mediastinum . Which of the following extracellular virulence factors most likely enables the causal organism to evade phagocytosis ?
;

A ) Alginate
B ) Glucuronoxylomannan
C ) Hyaluronic acid
D ) Polyglutamic acid
E ) Pollyribitol phosphate

o
Previous
o
Next
£5
Lab Values
e
Calculator
©
Review
&
Help Pause
Exam Section 1: Item 33 of 50 National Board of Medical Examiners ^ Time Remaining:
Mark Comprehensive Basic Science Self-Assessment 1 hr 13 min 28 sec

38. A 43 -year -old woman comes to the physician because of a 1 -year history of progressive ringing in her right ear. She also has felt dizzy while exercising. Neurologic examination shows dysmetria of the right upper and lower extremities. Muscle strength and
somatosensory function testing of all extremities shows no abnormalities. Audiometry shows moderate hearing loss in the right ear. An MRI of the brain is most likely to show a mass compressing which of the following labeled structures in the photograph of a
cross section of the brain stem?

r :
* .-
X '
*
\ s

P*
ft %

O A)
O B}
O C)
O D)
O E}
O F)

o
Previous
o
Next Lab Values Calculator Review
m
Help
o
Pause
Exam Section 1: Item 39 of 50 National Board of Medical Examiners ^ Time Remaining:
Mark Comprehensive Basic Science Self-Assessment 1 hr 13 min 2 sec

39. An investigator is designing a study to compare a new behavioral program for attention-deficit/ hyperactivity disorder (ADHD ) with the standard behavioral modification program for this disorder. Because ADHD is more common among boys: girls and boys are
randomized into the two treatment groups separately. Which of the following types of treatment allocation is most likely being used in this study?

A ) Alternation
CJ B ) Open
O C ) Outcome-adapted
D ) Simple random assignment
CJ E } Stratification

o
Previous
o
Next
E
Lab Values Calculator
m
Review
r
Help *r*
Pause
Exam Section 1: Item 40 of 50 National Board of Medical Examiners ^ Time Remaining:
Mark Comprehensive Basic Science Self-Assessment 1 hr 12 min 59 sec

40. A 69-year -old man with hepatic disease and increased prothrombin time is scheduled to undergo a left hip replacement operation . PreoperativeI y. the patient receives a transfusion of 1000 mL of fresh frozen plasma . Two hours later he develops chills,
abdominal cramps, dyspnea vomiting and diarrhea. The most likely cause of this patient's reaction is a genetic deficiency in which of the following immune mediators ?
; ,

O A ) C3
O B } IgA
O C ) Interferon gamma
D ) lnterleukin -2 ( IL-2)
C_ E ) Perforin

o
Previous
o
Next
E
(
Lab Values
!
I
Calculator
A
Review
&
Help
tl*
Pause
Exam Section 1: Item 41 of 50 National Board of Medical Examiners ^ Time Remaining:
Mark Comprehensive Basic Science Self-Assessment 1 hr 12 min 57 sec

41 . A 55 -year -old woman is scheduled to undergo tnansvaginaI hysterectomy and oophorectomy for dysfunctional uterine bleeding . During the procedure: the uterus must be separated from all surrounding pelvic structures . Identification and incision of which of
the following structures that attaches to the cervical region and extends posteriorly is most appropriate in this patient?

O A ) Mesometrium
B ) Mesosalpinx
C ) Mesovaiiium
D ) Ovarian ligament
E ) Round ligament of the uterus
F ) Uterosacral ligament

o
Previous
o
Next
E
(
Lab Values
I!
Calculator
A
Review
r
Help
&
Pause
Exam Section 1: Item 42 of 50 National Board of Medical Examiners ^ Time Remaining:
Mark Comprehensive Basic Science Self-Assessment 1 hr 12 min 55 sec

42 . An 18-year - old man with acute lymphoblastic leukemia comes to the physician because of a 3- day history of intermittent fever and left- sided chest pain. He is in the fifth week of induction chemotherapy
consisting of asparaginase , daunorubicin , prednisone , and vincristine. His absolute neutrophil count has been less than 50G/rnm 3 for the past month . His temperature is 39.2 X (1 Q2.6 CF ). Physical examination
shows dullness to percussion and decreased breath sounds on the left side . A chest x -ray shows a left lower lobe infiltrate and a moderate pleural effusion on the left . A photomicrograph of a periodic acid-Schiff
stain of pleural fluid obtained via thoracentesis is shown . Which of the following is the most appropriate pharmacotherapy for this patient?

A ) Amphotericin B
6 } Ertapenem
O G ) Ganciclovir
D ) Infliximab
E ) Rituximab
t ~
• M

J
Kki* 14
1
* * ay
{
4
*
*
;
t

Ml h

o
Previous
o
Next Lab Values
e
Calculator
©
Review
&
Help Pause
Exam Section 1: Item 43 of 50 National Board of Medical Examiners ^ Time Remaining:
Mark Comprehensive Basic Science Self-Assessment 1 hr 12 min S3 sec

43. A 53-year -ald woman comes to the physician because of a 6 -month history of increasingly severe abdominal discomfort . She is 170 cm ( 5 ft 7 in) tall and weighs 104 kg (230 lb) ; BMI is 36 kg /m 2 Abdominal examination shows fullness and tenderness of the
right lower quadrant that increases with coughing . A CT scan of the abdomen is shown; the arrow indicates an abnormality. Which of the following structures forms the medial border of this abnormality?

O A ) Arcuate line
B ) Inguinal ligament
O C ) Rectus abdominis
D ) Round ligament
! E ) Transversus abdominis

o
Previous
o
Next
tS
Lab Values Calculator
m
Review Help Pause
Exam Section 1: Item 44 of 50 National Board of Medical Examiners^ Time Remaining:
Mark Comprehensive Basic Science Self-Assessment 1 hr 12 min 51 sec

44 . A 2 - month-old boy is brought to the physician by his mother for a well-child examination . The mother tells the physician that her son has been gazing at her more directly. He also indicates delight with gestures when he sees her and shows signs of distress
when he cannot see her. Which of the following best describes this patient's recent behavior ?

A ) Attachment
B ) Enrrieshment
C ) Manipulation
D ) Merging
[ E ) Separation anxiety

o
Previous
o
Next Lab Values
I!
Calculator
A
Review
m
Help
o
Pause
Exam Section 1: Item 45 of 50 National Board of Medical Examiners ^ Time Remaining:
Mark Comprehensive Basic Science Self-Assessment 1 hr 12 min 48 sec

45. A 53-year -old man conies to the physician for a health maintenance examination . He does not smoke . He is 1 SO cm (5 ft 11 in ) tall and weighs 95 kg (210 lb) ; BMI is 29 kg /m 2 His blood pressure is 140/ 92 mm Hg. Physical examination shows no other
abnormalities . Serum studies show :
Cholesterol, total 240 mg/dL
HDL- cholesterol 35 mg/dL
LDL -cholesterol 170 mg/dL
Triglycerides 175 mg/dL

In addition to appropriate pharmacotherapy, it is most appropriate for this patient to decrease his dietary intake of which of the following types of fatty acids ?

O A) cjs-Unsaturated

B ) Monoun saturated
C ) Omega-3
D ) Omega-6
E ) Omega -9
F ) fra/is-Unsaturated

o
Previous
o
Next Lab Values Calculator
m
Review
c*
Help
it*
Pause
Exam Section 1: Item 46 of 50 National Board of Medical Examiners ^ Time Remaining:
Mark Comprehensive Basic Science Self-Assessment 1 hr 12 min 45 sec

46. A 1 - month-old male newborn is brought to the emergency department by his mother because of a 5-day history of vomiting after feedings. His mother says the vomiting is forceful, and appears to contain only formula . She says he seems eager to feed after
each episode . He appears irritable. He is at the 25th percentile for length and weight . His temperature is 37 6DC (99.7T ), pulse is 140/ min : respirations are 24/min and blood pressure is 30/ 40 mm Hg . Physical examination shows decreased skin turgor.
:

Which of the following sets of serum findings (in mEq /L ) is most likely in this newborn at this time?

Na + ( N=134~146 ) K + ( N=3-6 ] d- (N=95-115) HCO -f ( N=20-28 )


O A) 132 32 90 37
O B) 132 32 100 25
O C) 132 4.0 00 35
O D) 135 3.5 110 14
O E) 133 4.6 00 16
O F) 133 4.6 110 23

O O * ® r r
Previous Next Lab Values Calculator Review Help Pause
Exam Section 1: Item 47 of 50 National Board of Medical Examiners ^ Time Remaining:
Mark Comprehensive Basic Science Self-Assessment 1 hr 12 min 43 sec

47. A 33-year -old woman with HIV infection is brought to the emergency department 30 minutes after she had a generalized tonic-clonic seizure . She also has a 2 -month history of daily headaches. Physical examination shows no signs of meningismus . Muscle
strength is 3/5 in the left upper extremity and 5 /5 in the right upper extremity. Her CD 4+ T-lymphocyte count is 22/mm 3 (N>5 G 0) , and plasma HIV viral load is 50 000 copies/mL . A CT scan of the head shows a 3- cm lesion in the right cerebral cortex . Serologic
?

studies show a positive IgG antibody titer to Toxoplasma gondii . Treatment with pyrimethamine and sulfadiazine is initiated . During the next 2 weeks she has three additional seizures . Two weeks after starting antibiotic therapy a CT scan of the head shows
;

that the lesion has increased to 3.5 cm.. Which of the following is the most likely cause of this mass?

O A ) Bacterial abscess
o B ) Cerebral toxoplasmosis
C) C ) Glioblastoma
o D ) Metastatic disease
o E ) Non Hodgkin lymphoma
-

o
Previous
o
Next
IE
Lab Values
e
Calculator
o
Review Help Pause
Exam Section 1: Item 43 of 50 National Board of Medical Examiners ^ Time Remaining:
Mark Comprehensive Basic Science Self-Assessment 1 hr 12 min 40 sec

48. A 20 -year -old woman comes to the physician because of a 1 -year history of intermittent low-grade fever fatigue and joint pains. One week ago she had a severe exacerbation of these symptoms after a trip to the beach : she also developed a red rash even
: ; :

on areas that were not exposed to sun . Her temperature is 38.3°C (101 aF) Physical examination shows an erythematous rash over the malar areas, trunk , and upper extremities; oral ulcerations: 1-cm axillary lymph nodes bilaterally ; and mild swelling and
tenderness of the joints of the hands . Serologic studies show an antinuclear antibody titer of 1 :2 GG. Further studies are most likely to show a mutation of a gene encoding which of the following proteins ?

O A ) C1q
B ) C -reactive protein
C ) Interferon gamma
D ) Mannose -binding lectin
E ) Serum amyloid A

o
Previous
o
Next
It?
Lab Values
*
Calculator
®
Review
f
Help Pause
Exam Section 1: Item 49 of 50 National Board of Medical Examiners ^ Time Remaining:
Mark Comprehensive Basic Science Self-Assessment 1 hr 12 min 38 sec

49. A 50 -year -old man with type 2 diabetes mellrtus comes to the physician for a follow-up examination . Current medications include long-acting insulin and metformin . His vital signs are within normal limits. Physical examination shows no abnormalities. His
hemoglobin Alc is 7.9% . Treatment with a third drug that promotes release of endogenous insulin is initiated to improve glucose control . This drug is most likely which of the following?

O A ) Acarbose
B ) Miglitol
O C ) Pioglitazone
D ) Pramlintide
C_ E ) Sitagliptin

o
Previous
o
Next
(E
Lab Values
I!
Calculator
A
Review
r
Help
&
Pause
Exam Section 1: Item SO of 50 National Board of Medical Examiners ^ Time Remaining:
Mark Comprehensive Basic Science Self-Assessment 1 hr 12 min 36 sec

50. A 57 -year -old man is brought to the emergency department because of a 6- hour history of acute lower abdominal and flank pain . He also has a 6-week history of fatigue and generalized muscle and bone pain . He appears ill and changes positions to relieve
the pain when it occurs. Physical examination shows a 0 5-cmp nontender movable mass in the left side of the neck . Abdominal examination shows decreased bowel sounds; there is no rebound tenderness. Laboratory studies show:
Serum
Ca 2+ 12.8 mg/dL
Urea nitrogen 25 mg/dL
Creatinine 1.5 mg/dL
Urine
RISC 3-5/hpf

An x-ray of the abdomen shows a calculus in the proximal left ureter. Which of the following is the most likely underlying cause of this patient’s condition?

O A ) Parathyroid adenoma
o B ) Parathyroid carcinoma
o C ) Parathyroid hyperplasia
o D ) Thyroid follicular adenoma
o E ) Thyroid nodular hyperplasia
o F ) Thyroid papillary carcinoma

Previous Next
It?
Lab Values
*
Calculator
o
Review Help Pause
Exam Section 2: Item 1 of 50 National Board of Medical Examiners^ Time Remaining:
9 Mark Comprehensive Basic Science Self-Assessment 1 hr 14 min 56 sec

1 . Lysosomes present in hepatocytes of patients with mucolipidosis II ( l -cell disease ) contain abnormal enzyme activities resulting from an inability of these cells to do which of the following?

0 A ) Glycosylate secretory proteins


CJ B } Recycle lysosomes
O C ) Recycle mannose 6-phosphate receptors from the plasma membrane
D ) Synthesize lysosomal enzymes
E ) Synthesize mannose 6-phosphate residues on proteins

- © ©
Next Lab Values Calculator Review Help Pause
Exam Section 2 : Item 2 of 50 National Board of Medical Examiners ^ Time Remaining:
Mark Comprehensive Basic Science Self-Assessment 1 hr 14 min 53 sec

2 . A woman who weighs 70 kg (154 lb) is participating in a research study. Her hematocrit is 40%, and her plasma volume is 3 L . The subject' s total blood volume (in L) is closest to which of the following?

.... A) 4
O B} 5
O C) 6
O D> 7
O E} 8

o
Previous
o
Next
tS
Lab Values
*
Calculator
m
Review
c*
Help
it*
Pause
Exam Section 2 : Item 3 of 50 National Board of Medical Examiners^ Time Remaining:
Mark Comprehensive Basic Science Self-Assessment 1 hr 14 min 51 sec

3 . The solid curve represents the normal oxygen-hemoglobin dissociation curve . The dashed curve is consistent with an increase in which of the following? & 100 -
ao - /
C A ) 2 3- 6isphosphoglycerate concentration
; C5
#
t
I

6 } Body temperature 1 60 -
Jfr t
#
f
i
=
1
40 -
(_) C ) Fetal hemoglobin concentration 15
o I
*
f

CD f
D ) CO concentration o 20
E -
<D
O E } Pco 2 0
0 20 40 60 80 100 120
T

PO 2 (mm Hg)

O O if!
Previous Next Lab Values Calculator Review Help Pause
Exam Section 2 : Item 4 of 50 National Board of Medical Examiners ^ Time Remaining:
Mark Comprehensive Basic Science Self-Assessment 1 hr 14 min 48 sec

4 . A 30 -year-old woman has polyhydramnios at 25 weeks ' gestation . The fetus has an anatomically normal esophagus. Which of the following abnormalities is the most likely cause of the polyhydramnios ?

0 A ) Anencephaly
B ) Caudal dysplasia
0 C ) Lumbar myelomeningocele
D ) Renal agenesis
0 E } Urethral obstruction

o
Previous
o
Next Lab Values
I!
Calculator
0
Review Help Pause
Exam Section 2 : Item 5 of 50 National Board of Medical Examiners^ Time Remaining:
Mark Comprehensive Basic Science Self-Assessment 1 hr 14 min 46 sec

5 . A 45 -year-old man has shortness of breath . He has never smoked cigarettes . There is no evidence of cardiac disease. X-rays of the chest show bullous air spaces in the lower lobes . The pathophysiologic basis of this process is most likely to be a deficiency of
which of the following?

O A ) Ceruloplasmin
B ) Epidermal growth factor
C ) Ferritin
D ) Plasminogen
E ) Protease inhibitor

o
Previous
o
Next
lE
Lab Values
*
Calculator
m
Review Help Pause
Exam Section 2 : Item 6 of 50 National Board of Medical Examiners^ Time Remaining:
Mark Comprehensive Basic Science Self-Assessment 1 hr 14 min 44 sec

6 . Which of the following curves best describes variation of follide-stimiulating hormone concentration during the menstrual cycle ?

m
c
O
§ IC
o
JZ
O
C.
Q

c
U
c
o
u

1 14 29
Day of menslrual cycle

... A)
O B)
O C)
)
O E)

o
Previous
r
Next Lab Values Calculator
A
Review
m
Help
o
Pause
Exam Section 2 : Item 7 of 50 National Board of Medical Examiners ^ Time Remaining:
Mark Comprehensive Basic Science Self-Assessment 1 hr 14 min 41 sec

7 . A 60 -year-old man has two-pillow orthopnea: severe dyspnea and edema of the lower extremities . The onset of these symptoms was preceded by an episode of prolonged substernal chest pain 5 days ago. Which of the following additional findings on
;

examination of the neck is most consistent with these findings ?

A ) Bifid carotid pulse


B ) Carotid bruit best heard at the carotid bifurcation
O C ) Jugular " cannon waves' 1

D ) Jugular venous pressure of 12 mm Hg


' _ ! E ) Slow-rising decreased-volume carodd pulse
,

o
Previous
o
Next Lab Values
I!
Calculator
A
Review
m
Help
o
Pause
Exam Section 2 : Item 8 of 50 National Board of Medical Examiners ^ Time Remaining:
Mark Comprehensive Basic Science Self-Assessment 1 hr 14 min 39 sec

8 . The breakdown of dipeptides and tripeptides to free amino acids takes place primarily in which of the following areas in the gastrointestinal tract?

0 A ) Intestinal mucosa
B } Lumen of the duodenum
0 C ) Lumen of the large intestine
D ) Lumen of the stomach
E ) Mouth

o
Previous
o
Next Lab Values
*
Calculator
m
Review Help Pause
Exam Section 2 : Item 9 of 50 National Board of Medical Examiners^ Time Remaining:
Mark Comprehensive Basic Science Self-Assessment 1 hr 14 min 37 sec

9 . An 18 -year- old man has a 1 -hour history ofwheezing chest tightness , and shortness of breath . He weighs 60 kg (132 lb ) . Minute ventilation is 14 L/ min (N=5-10 L/min). Which of the following sets of arterial blood gas findings is most likely present in this
;

patient?

Pco 2 Po 2
pH { mm Hg ) ( mm Hg )
O A) 7.30 70 50
O B) 7.30 30 103
O C) 7.40 30 75
) 7.40 39 103
O E) 7.48 30 75
O F) 7.48 70 50

O O E
( !
I A r &
Previous Next Lab Values Calculator Review Help Pause
Exam Section 2: Item 10 of 50 National Board of Medical Examiners ^ Time Remaining:
Mark Comprehensive Basic Science Self-Assessment 1 hr 14 min 35 sec

10. A 45 -year-old man with mild hypertension uses a nasal spray for allergic rhinitis and nasal congestion . Four days later the spray no longer relieves his symptoms and his blood pressure is increased. Which of the following best describes the action of this
,

drug?

3 A ) arAdrenergic receptor agonist


3 B} p Adrenergic receptor antagonist
^
O C ) Histaminergic ( 3) receptor antagonist
D ) Muscarinic receptor agonist
3 E ) Prostaglandin ( PGE 1) receptor agonist

o
Previous
o
Next
tS
Lab Values Calculator
m
Review
f*
Help
ft*
Pause
Exam Section 2: Item 11 of 50 National Board of Medical Examiners^ Time Remaining:
Mark Comprehensive Basic Science Self-Assessment 1 hr 14 min 33 sec

11 A 36 -year - old man who is seropositive for HIV has had fever and shortness of breath for 2 days. An x-ray of the chest shows a diffuse i nterstitial i nfiltrate. AIung biopsy is shown . Which of the following
organisms is most likely involved?

0 A ) Cryptococcus neoformans
6 } Cytomegalovirus
C C ) Epstein - Barr virus
D ) Histoplasma capsulatum
E ) Influenzavirus A
F } Mycobacterium avium-intracellulare
(J G ) Pneumocystis jirovecii (formerly P. carinii )

*9

o
Previous
o
Next
ts
Lab Values Calculator Review Help Pause
Exam Section 2: Item 12 of 50 National Board of Medical Examiners ^ Time Remaining:
Mark Comprehensive Basic Science Self-Assessment 1 hr 14 min 30 sec

12 . A study is conducted on patients with asthma to compare a new asthma treatment with a placebo. After 12 weeks of treatment the mean (± standard error of the mean ) scores are 2.8 ± 0.1 in the treated group (n = 103) and 3.1 ± 0.1 in the placebo group (n =
100). The p-value is 0.035. Which of the following best describes the meaning of the p-value ?

O A ) 3.5% of the treated patients had asthma scores lower (better ) than the mean score of the controls
o B ) 96.5% of the treated patients had asthma scores lower (better) than the mean score of the controls
o C ) The drug worked in 3.5% of the patients
o D ) The drug worked in 96.5% of the patients
o E ) If the drug really does not work there is a 3.5% chance of finding an average difference of 0.3 or greater
;

o F ) If the drug really does not work there is a 96.5% chance of finding an average difference of 0.3 or greater
;

Previous Next Lab Values


*
Calculator
o
Review Help Pause
Exam Section 2: Item 13 of 50 National Board of Medical Examiners ^ Time Remaining:
Mark Comprehensive Basic Science Self-Assessment 1 hr 14 min 28 sec

13. A 7-day- old female newborn is brought to the physician because blood has been oozing from the umbilical area for more than 24 hours. She was born at home has had no previous medical treatment and has been breast-fed since birth . Which of the
,

following biochemical reactions is most likely defective in this patient?

C ) A ) Cross-linking of collagen and elastin


B ) Hydroxylation of proline
O C ) Post-translational carboxylation of glutamate residues
! _ D ) Synthesis of thymidylate

o
Previous
o
Next Lab Values Calculator
m
Review Help Pause
Exam Section 2: Item 14 of 50 National Board of Medical Examiners^ Time Remaining:
Mark Comprehensive Basic Science Self-Assessment 1 hr 14 min 26 sec

14 . A 29 -year-old man has a left testicular mass. A frozen section obtained during a planned orchiectomy shows a semiinoma . Which of the following provides the greatest risk factor associated with this condition?

0 A ) Alcohol abuse
B ) Cryptorchidism
0 C ) Mumps orchitis
D ) Varicocele
0 E ) Vasectomy

o
Previous
o
Next Lab Values
I!
Calculator
A
Review
m
Help
o
Pause
Exam Section 2: Item 15 of 50 National Board of Medical Examiners ^ Time Remaining:
Mark Comprehensive Basic Science Self-Assessment 1 hr 14 min 24 sec

15. A 45 -year-old man is brought to the physician by his parents because he is reclusive , has "weird ideas," and still wants to live at home. He is dressed in several layers of clothes . During the interview he is cooperative but he has poor eye contact and seems
;

uninterested in the interview. He says " I believe some people can read other people's minds. I carry these crystals to help me realize self- actualization . " He has no delusions or hallucinations . This patient most likely has which of the following personality
;

disorders ?

A ) Borderline
B ) Dependent
C ) Histrionic
D ) Narcissistic
E ) Schizoid
(_ F ) Schizotypal

o
Previous
o
Next Lab Values Calculator Review
c*
Help
it*
Pause
Exam Section 2: Item 16 of 50 National Board of Medical Examiners ^ Time Remaining:
Mark Comprehensive Basic Science Self-Assessment 1 hr 14 min 22 sec

16. In a sample of 100 individuals, the mean leukocyte count is 75DQ /mm 3. with a standard deviation of 1000/ mm 3 Assuming that the leukocyte counts in this population follow a normal ( gaussian) distribution : which of the following best represents the percentage
of time that a randomly selected individual will have a total leukocyte count lower than 5500/mm 3?

O A ) 1%
O B ) 2.5%
O C ) 5%
O D ) 10%
O E ) 16.5%
0 F ) 33.2%

o
Previous
o
Next
'zL
!

Lab Values Calculator


A
Review
&
Help
ti*
Pause
Exam Section 2: Item 17 of 50 National Board of Medical Examiners ^ Time Remaining:
Mark Comprehensive Basic Science Self-Assessment 1 hr 14 min 20 sec

17. A 45 -year-old man who works in a government building is brought to the emergency department because of a 2 - houir history of severe shortness of breath with obvious bronchoconstriction and cough productive of copious secretions. He also has had nausea ;

vomiting, and diarrhea. At least 20 other people in his office building were affected by similar symptoms at approximately the same time. Two of these coworkers died on the way to the hospital. Only a few of the affected employees ate together in the
government cafeteria that day. His respirations are 36/min . Physical examination shows excessive lacrimation . Which of the following is the most likely cause of the sudden disease outbreak?

A ) Anthrax infection
B ) Anticholinesterase poisoning
C ) Botulinum toxin poisoning
D ) Staphylococcus endotoxin poisoning
! _ E ) Viral pneumonitis and gastroenteritis

o
Previous
o
Next
(£5
Lab Values
e
Calculator
©
Review
&
Help Pause
Exam Section 2: Item 13 of 50 National Board of Medical Examiners ^ Time Remaining:
Mark Comprehensive Basic Science Self-Assessment 1 hr 14 min 17 sec

18. A 53 -year-old woman is brought to the emergency department because of a 2 -hour history of shortness of breath and chest pain that radiates to her back between the shoulder blades. Her respirations are 28/min. Physical examination shows diaphoresis. An
ECG shows no abnormalities Coronary angiography shows occlusion of the marginal branch of the left anterior descending coronary artery. A percutaneous coronary revascularization is done. Subsequent to the stent’s placement, her serum concentrations
of myocardial creatine kinase (CK- MB) and troponin I are increased. Which of the following mechanisms best explains these laboratory findings ?

A ) Cell shrinkage
B ) Formation of apoptotic bodies
C ) Liquefactive necrosis of the myocardium
D ) Membrane lipid peroxidation
! _ E ) Protease inactivation by cytoplasmic free calcium ions

o
Previous
o
Next
It?
Lab Values
*
Calculator
o
Review Help Pause
Exam Section 2: Item 19 of 50 National Board of Medical Examiners ^ Time Remaining:
Mark Comprehensive Basic Science Self-Assessment 1 hr 14 min 15 sec

19. A 42 -year-old man conies to the physician because of chronic abdominal pain for 3 months. He describes his symptom as boring epigastric pain that is occasionally exacerbated by eating . He is an investment banker and describes his job as extremely
stressful . Use of an over -the -counter histaminergic (HJ- receptor antagonist has mixed results. Vital signs are normal . Physical examination and upper endoscopy show no abnormalities . Which of the following is the most likely diagnosis ?

O A ) Gastric lymphoma
B ) Generalized anxiety disorder
O C ) Irritable bowel syndrome
D ) Major depressive disorder
! E ) Nonulcer dyspepsia

o
Previous
o
Next
tS
Lab Values Calculator
m
Review
c*
Help
it*
Pause
Exam Section 2: Item 20 of 50 National Board of Medical Examiners^ Time Remaining:
Mark Comprehensive Basic Science Self-Assessment 1 hr 14 min 13 sec

20. An investigator working in a large health care system wishes to determine the attitude of physicians toward drug screening in the workplace. To collect the information physicians in the health system are invited to respond to an e -mail survey that is distributed
on the health system's network . Which of the following most closely describes this type of sampling technique?

O A ) Convenience
B ) Random
O C ) Referral
C: D ) Stratified

o
Previous
o
Next
lE
Lab Values Calculator
m
Review
C*
Help
it*
Pause
Exam Section 2: Item 21 of 50 National Board of Medical Examiners ^ Time Remaining:
Mark Comprehensive Basic Science Self-Assessment 1 hr 14 min 11 sec

21. A 62 -year -old man conies to the physician because of a swollen , painful left great toe that awakened him from sleep this morning .. Examination of the toe shows erythema , swelling and exquisite tenderness. Laboratory studies show hyperuricemia . Treatment
,

with colchicine resolves the acute symptoms . Long -term therapy with allopurinol is begun . This treatment results in decreased urate biosynthesis. A diagram of purine synthesis is shown . Which of the following labeled steps is directly inhibited by allopurinol as
a result of this therapy?

Ribose 5- phosphate

A|
P hos phoribosylpyrop hasp h ate

B|
S- Phosphonbosylamine

j
C
Adenylosuccinate
D
AMP GMP

l
Hypoxanthine Xanthine
G t
Urate

.... A )

O B}
O c)
)
O E}
O F)
O G)

o
Previous
o
Next
E
(
Lab Values Calculator Review Help Pause
Exam Section 2: Item 22 of 50 National Board of Medical Examiners ^ Time Remaining:
Mark Comprehensive Basic Science Self-Assessment 1 hr 14 min 9 sec

22 . A 7G -year -old man conies to the physician with his wife because of a G - month history of abnormal behavior during sleep . His wife says "He suddenly punches at the air and yells as if he were acting out his dreams but he doesn't wake up. " As a result of his
; ;

agitation he has fallen on the floor several times Neurologic examination shows no abnormalities. Which of the following additional findings is most likely in this patient?
,

O A ) Absent muscle atonia during REM sleep


B ) Anterior cingulate mass lesion
O C ) Evidence of major depress i ve di sorder w it h p sychoti c features
D ) Left temporal lobe spikes on EEG
E ) Nocturnal hypoglycemia

o
Previous
o
Next
(E
Lab Values
!
I
Calculator
A
Review
r
Help
&
Pause
Exam Section 2: Item 23 of 50 National Board of Medical Examiners ^ Time Remaining:
Mark Comprehensive Basic Science Self-Assessment 1 hr 14 min 7 sec

23. A 25 -year -old woman comes to the physician because of a 9 -month history of fatigue, excessive sweating heat intolerance , palpitations , and nervousness; she has had a 9- kg (20 - lb) weight loss during this period. Her pulse is 105/m in. Physical examination
;

shows moist skin and a goiter. Which of the following is most likely decreased in this patient?

O A ) Iodide uptake by the thyroid gland


B ) Number of thyroid- stimulating hormone receptors
O C } Peripheral conversion of thyroxine (TJto triiodothyronine (TJ
D ) Recycling of intrathyroid iodide
E ) Thyrotropin - releasing hormone concentration

o
Previous
o
Next
E
(
Lab Values
I!
Calculator
A
Review
r
Help
&
Pause
Exam Section 2: Item 24 of 50 National Board of Medical Examiners ^ Time Remaining:
Mark Comprehensive Basic Science Self-Assessment 1 hr 14 min 5 sec

24 . A 25 -year -old woman comes to the physician 2 days after noticing a mass in her right axilla . She also has a 1 -week history of malaise, headaches , and night sweats . The patient says that she recently adopted a kitten and has sustained several bite and
scratch marks. Her temperature is 37.8 aC (1 GQCF) . Physical examination shows edema and tenderness of the right axillary lymph node . The skin over the node is erythematous, tough and warm. There are scratches and bite marks of various ages over the
,

upper extremities and hands . The result of an indirect fluorescent antibody test for Bartonella henselae is positive. A biopsy specimen of this lymph node is most likely to show which of the following histologic patterns ?

A ) Diffuse neutrophil infiltration


B ) Granulomas containing stellate microabscesses
C ) Large activated lymphocytes with occasional plasma cells
D ) Serous inflammation with abundant intravascular fibrin and few inflammatory cells
E ) Vascular proliferation with fibrosis

o
Previous
o
Next
It?
Lab Values
*
Calculator
o
Review Help Pause
Exam Section 2: Item 25 of 50 National Board of Medical Examiners ^ Time Remaining:
Mark Comprehensive Basic Science Self-Assessment 1 hr 14 min 3 sec

25. A 3-year-old boy is brought to the emergency department by his mother 1 hour after having a generalized tonic-clonic seizure . He has had frequent vomiting and progressive stupor during the past 24 hours. His mother says that she gave him aspirin 2 days
ago because of a temperature of 39.5 X ( 103.1 aF). Physical examination shows hepatomegaly with no jaundice. Serum studies show an increased ammonia concentration . and increased AST and ALT activities . Dysfunction of which of the following is the
most likely cause of this patient's disorder ?

A ) Glutamate transporters
B ) Golgi complex
C ) Microtubules
D ) Mitochondria
E ) Rough endoplasmic reticulum

o
Previous
o
Next
lE
Lab Values Calculator
m
Review
C*
Help
it*
Pause
Exam Section 2: Item 26 of 50 National Board of Medical Examiners ^ Time Remaining:
Mark Comprehensive Basic Science Self-Assessment 1 hr 14 min 1 sec

26. A 42 -year -old man conies to the physician for an examination prior to beginning an exercise program at a local health club . He has smoked 1 pack of cigarettes daily for 20 years . He is 183 cm (6 ft } tall and weighs 70 kg (155 lb ); BMI is 21 kg /m 2 His blood
pressure is 130/S 0 mm Hg. Physical examination shows no other abnormalities. Serum studies show :
Cholesterol total 350 mg/dL
HDL-cholesterol 40 mg/dL
LDL-cholesterol 280 mg/dL
Triglycerides 150 mg/dL

Which of the following defects is the most likely cause of this patient's increased serum LDL -cholesterol concentration?

1} A ) Decreased ability to convert cholesterol to bile acids


o B ) Decreased clearance of cholesterol by the liver
o C ) Increased absorption of cholesterol in the gut
o D ) Increased production of cholesterol by adipose tissue
o E ) Increased production of cholesterol by the liver

o
Previous
o
Next
It?
Lab Values
*
Calculator
®
Review
f
Help Pause
Exam Section 2: Item 27 of 50 National Board of Medical Examiners ^ Time Remaining:
Mark Comprehensive Basic Science Self-Assessment 1 hr 13 min 59 sec

27. A 19-year-old man is admitted to the hospital following a motor vehicle collision . Physical examination shows a penetrating wound to the right cerebral cortex with comp ete paralysis of the left lower extremity, fracture of the right mid humerus with severing of
the radial nerve: and a fracture of the right tibia . Treatment includes cast immobiliza:ion of the right upper extremity and right knee and ankle. After 10 weeks, the casts are scheduled to be removed from the right upper and lower extremities. At this point the
,

deep tendon reflex is most likely to be strongest in which of the following locations in this patient?

A ) Left Achilles tendon


B ) Left biceps tendon
C ) Right braclhioradialis tendon
! _ D ) Right patellar tendon

o
Previous
o
Next
(E
Lab Values
I!
Calculator
A
Review
r
Help
&
Pause
Exam Section 2: Item 23 of 50 National Board of Medical Examiners ^ Time Remaining:
Mark Comprehensive Basic Science Self-Assessment 1 hr 13 min 57 sec

28. A 65 -year -old man conies to the physician 2 weeks after noticing a lesion on his neck . He is an avid golfer, and he always wears a hat but does not always use sunscreen while playing . Examination of the neck shows a 1G-mm, pearly pink raised nodule . This
;

patient' s condition most likely originated in which of the following?

A ) Epidermis
B ) Papillary dermis
O C ) Reticular dermis
D ) Subcutaneous tissue
E ) Deep fascia

o
Previous
o
Next Lab Values
I!
Calculator
A
Review
m
Help
o
Pause
Exam Section 2: Item 29 of 50 National Board of Medical Examiners ^ Time Remaining:
Mark Comprehensive Basic Science Self-Assessment 1 hr 13 min 55 sec

[ X]
V=
K„ + [ X]
29. The formula shown describes the rate of metabolism of Drug X where v is the rate of metabolism, V max is the maximal rate of metabolism, and Michaelis constant [K ] is the plasma concentration at one half the maximum rate of metabolism. If the
^
concentration of Drug X is 100 times greater than the Kmof the metabolic enzyme for Drug X . which of the following conclusions is most appropriate ?

A ) First- order kinetics governs the rate of the reaction


B ) Further metabolism of Drug X is inhibited
O C ) Metabolism of Drug X involves covalent bonding of the enzyme
D ) The velocity of the reaction is proportional to the concentration of Drug X
E } A zero-order rate of metabolism governs the reaction

O O m
Previous Next Lab Values Calculator Review Help Pause
Exam Section 2: Item 30 of 50 National Board of Medical Examiners ^ Time Remaining:
Mark Comprehensive Basic Science Self-Assessment 1 hr 13 min 52 sec

30. An S 2 - year - old woman is admitted to the hospital for treatment of a small right pleural effusion . Physical examination shows dullness to percussion at the level of the 7th rib and below on the right . A thoracentesis is done. The patient is sitting up, supported by
leaning against a bedside table . After preparing the skin and delivering a local anesthetic into which of the following locations on the right is it most appropriate to insert the catheter ?

O A ) Above the 5th rib in the midscapular line


B ) Above the 5th rib just to the right of the sternum
o C ) Above the 7th rib in the paravertebral region
o D ) Above the 9th rib in the midscapular line
o E } Below the 5th rib in the midaxillary line
o F ) Below the 5th rib in the midclavicular line
o G ) Below the 9th rib in the midscapular line

o
Previous
o
Next Lab Values
*
Calculator
0
Review
9
Help
0
Pause
Exam Section 2: Item 31 of 50 National Board of Medical Examiners ^ Time Remaining:
Mark Comprehensive Basic Science Self-Assessment 1 hr 13 min 42 sec

31. An investigator studying type 2 diabetes discovers an inhibitor of uncoupling protein-2 that stimulates insulin secretion from mouse pancreatic is et cells. This inhibitor most likely increases insulin secretion by which of the following actions ?

0 A ) Inactivating a tyrosine kinase


B ) Increasing glucokinase activity
0 C ) Increasing glucose transporter- 2 ( GLUT-2 ) expression
D ) Increasing the ATP : ADIP ratio
E ) Inhibiting the target of rapamycin (TOR ) protein kinase
F ) Stimulating the hydrolysis of phosphoinositide

o
Previous
o
Next
(E
Lab Values
I!
Calculator
A
Review
r
Help
&
Pause
Exam Section 2: Item 32 of 50 National Board of Medical Examiners ^ Time Remaining:
Mark Comprehensive Basic Science Self-Assessment 1 hr 13 min 30 sec

32 . A 43-year -old woman , gravida 2 , para 1. at 10 weeks' gestation conies to the physician for an initial prenatal visit . Physical examination shows a uterus consistent in size with a 10 -week gestation . This patient is at increased risk for a child with Down
syndrome if she has which of the following sets of ultrasound and serum findings?

Fetal Ultrasound Maternal Serum


Nuchal P reg n ancy -associate d Human Chorionic
Translucency Plasma Protein Gonadotropin
O A)
O B) ,

O C) *
i
D) *
i
O E) , ,

0 F)
- - i

o
Previous
o
Next
‘zL!
Lab Values Calculator
A
Review
r
Help
&
Pause
Exam Section 2: Item 33 of 50 National Board of Medical Examiners^ Time Remaining:
Mark Comprehensive Basic Science Self-Assessment 1 hr 13 min 28 sec

33. An investigator is studying a tumor cell line in an experimental animal model in which the expression of class I MHC is completely inhibited . The cell line is assayed as a target for cell - mediated immune response by CD 3+ T lymphocytes , CD 4+ T lymphocytes ;

or natural killer cells. Which of the following sets of findings is most likely ( + = killing : - = no killing }?

CDS + T Lymphocytes CD4+ T Lymphocytes Natural Killer Cells


O A) + + +

O B) + +

O C) +

) +

O E) +

O F)

o
Previous
o
Next
E
(
Lab Values
!
I
Calculator
A
Review
&
Help
tl*
Pause
Exam Section 2: Item 34 of 50 National Board of Medical Examiners ^ Time Remaining:
Mark Comprehensive Basic Science Self-Assessment 1 hr 13 min 25 sec

34 . A SS -year -old man with alcoholism comes to the physician because of a 2 -month history of increasingly severe stomach pain and increased volumes of foul- smelling stool : he also has had a 9-kg (20 - lb ) weight loss during this period .. He has a history of
multiple visits to the emergency department because of severe abdominal pain . He has consumed one bottle of red wine daily for 5 years. His temperature is 37DC (98 6DF ) Physical examination shows epigastric tenderness . His fasting serum glucose
concentration is 150 mg/dL. A CT scan of the abdomen shows pancreatic calcifications. The most likely cause of this patient’s current symptoms is a decrease in which of the following?

A ) Bile acid synthesis


B ) Colonic bacteria
C ) Duodenal pH
D ) Fecal elastase
! _ E ) 7a- Hydroxylase activity

o
Previous
o
Next
It?
Lab Values
*
Calculator
®
Review
r
Help
r
Pause
Exam Section 2: Item 35 of 50 National Board of Medical Examiners^ Time Remaining:
Mark Comprehensive Basic Science Self-Assessment 1 hr 13 min 24 sec

35. In an experiment on ion transport in epithelial cells isolated from various regions of the nephron hydrochlorothiazide inhibits Na + entry into one group of cells . These cells were most likely obtained from which of the following regions ?
;

0 A ) Cortical collecting tubule


B ) Distal convoluted tubule
0 C ) Proximal tubule
D ) Thick ascending limb of the loop of Henle
0 E ) Thin descending limb of the loop of Henle

o
Previous
o
Next
E
(
Lab Values
!
I
Calculator
A
Review
r
Help
&
Pause
Exam Section 2: Item 36 of 50 National Board of Medical Examiners ^ Time Remaining:
Mark Comprehensive Basic Science Self-Assessment 1 hr 13 min 21 sec

36. A 43-year -old man conies to the emergency department because of a 1 -year history of low back pain . Before the examination , the patient says , nMy physician is arrogant and insensitive. He never returns my phone calls , I always have to wait forever to be
seen the tests he orders are painful and unnecessary, and he can never tell me what is causing my back pain or how to treat it . ' Which of the following is the most appropriate response by the emergency department physician about this patient’s complaints ?
,
1

O A ) Reassure the patient that his physician's behavior is not unusual and that low back pain can be difficult to assess
B ) Encourage the patient to make an appointment with his physician to communicate his concerns
O C ) Explain to the patient how he may register a complaint against his physician with the state medical licensing board
D ) Provide the patient with a copy of the Patients Bill of Rights
1

' _ ! E ) Telephone the patient’s physician to make him aware that the patient is very dissatisfied with treatment

o
Previous
o
Next
It?
Lab Values
*
Calculator
o
Review Help Pause
Exam Section 2: Item 37 of 50 National Board of Medical Examiners ^ Time Remaining:
Mark Comprehensive Basic Science Self-Assessment 1 hr 13 min 19 sec

37. A 5-month-old boy is brought to the physician by his parents because of a 1 -week history of weakness . His parents are first cousins. Physical examination shows pallor jaundice, and hepatosplenomegaly. His
hemoglobin concentration is 6 g/ dL , hematocrit is 19%, and serum bilirubin concentration is 4.2 mg /dL (mostly indirect } . A photomicrograph of a peripheral blood smear is shown. Hemoglobin (Hb ) electrophoresis shows
HbF: no HbA, and an increased concentration of HbA 2. Which of the following is the most likely molecular cause of this patient' s condition?

C A } Deletion of one or more a - globin genes


B } Deletion of one or more p - globin genes
C C ) Mutation of the a -globin gene inherited from each parent
D ) Mutation of the a -globin gene inherited from one parent
E ) Mutation of the |3-globin gene inherited from each parent
F ) Mutation of the |3 -globin gene inherited from one parent

o
Previous
o
Next Lab Values
*
Calculator
®
Review Help
^
Pause
Exam Section 2: Item 33 of 50 National Board of Medical Examiners ^ Time Remaining:
Mark Comprehensive Basic Science Self-Assessment 1 hr 13 min 17 sec

38. A 63-year -old man with alcoholism is diagnosed with hepatic encephalopathy. Treatment with lactulose therapy via nasogastric tube is begun . The effectiveness of this intervention requires which of the following mechanisms?

0 A ) Absorbed lactulose binding alcohol in the blood decreasing blood alcohol concentrations
;

B ) Lactulose binding of ingested alcohol within the colonic lumen for excretion
0 C ) Metabolism to glucose and galactose increasing serum glucose concentrations
;

D ) Osmotic diarrhea , flushing out ingested alcohol from the gut


0 E ) Trapping of ammonia in the colon by acidic metabolites of lactulose

o
Previous
o
Next
lE
Lab Values Calculator
m
Review Help Pause
Exam Section 2: Item 39 of 50 National Board of Medical Examiners ^ Time Remaining:
Mark Comprehensive Basic Science Self-Assessment 1 hr 13 min 15 sec

39. A 23 -year -old woman , gravida 4 , para 0 . aborts 3 , with systemic lupus erythematosus comes to the physician at 12 weeks ' gestation for an initial prenatal examination . She is concerned that she will not be able to carry her current pregnancy to term . The
presence of which of the following is most likely increasing the risk for complications during pregnancy in this patient?

O A ) Antimitochondrial antibodies
B ) Antineutrophil cytoplasmic antibodies
O C ) Antiphospholipid antibodies
D ) Antistreptolysin O antibodies
E ) Rheumatoid factor

o
Previous
o
Next Lab Values Calculator
m
Review Help Pause
Exam Section 2: Item 40 of 50 National Board of Medical Examiners ^ Time Remaining:
Mark Comprehensive Basic Science Self-Assessment 1 hr 13 min 13 sec

40. A 35 -year -old woman comes to the emergency department because of abdominal cramps nausea and vomiting for 3 hours. Her pulse is 106/min and regular and blood pressure is 140/ 96 mm Hg . Physical examination shows rhinorrhea excessive
; ,
:

lacrimation . and diaphoresis. There is piloerection over most of the skin . The pupils are dilated, equal in size: and responsive to direct and indirect light . There is a resting tremor involving both upper extremities . During the examination , the patient is restless
and yawns constantly. This patient is most likely experiencing withdrawal symptoms from which of the following substances?

A ) Cannabis
B ) Cocaine
C ) Diazepam
D ) Ethanol
E ) Heroin
(_ F ) Phenobarbital

o
Previous
o
Next
(£5
Lab Values
e
Calculator
©
Review
&
Help Pause
Exam Section 2: Item 41 of 50 National Board of Medical Examiners ^ Time Remaining:
Mark Comprehensive Basic Science Self-Assessment 1 hr 13 min 11 sec

41 . A 55 -year -old woman comes to the physician because of a 3 -day history of persistent right shoulder pain . She began a weight-training program 6 weeks ago. She says that the pain intensified after she increased the amount of weight that she had been lifting
above her head . She rates the pain as 8 on a 10- point scale . Examination of the right shoulder shows point tenderness just lateral to the acromion , over the humeral head . Passive motion of the shoulder is full. Pain is reproduced with resisted abduction of the
shoulder when the shoulder is abducted 90 degrees and the arm is giving the "thumbs down" sign . Sensation is intact over the right upper extremity. Which of the following tendons is most likely injured in this patient?

A ) Deltoid
B ) Infraspinatus
C ) Subscapularis
o D ) Supraspinatus
o E ) Teres major
o F ) Teres minor

o
Previous
o
Next
It?
Lab Values
*
Calculator
®
Review
r
Help
r
Pause
Exam Section 2: Item 42 of 50 National Board of Medical Examiners ^ Time Remaining:
Mark Comprehensive Basic Science Self-Assessment 1 hr 13 m in 9 sec

42 . A 35 -year -old woman comes to the physician in the spring because of a 2-week history of tearing and itching of her eyes . She has a 10- year history of similar symptoms during this time of year. She attributes her condition to an allergy to pollen and grasses.
Her symptoms improve when she takes an over-the-counter antihistamine . The antihistamine therapy most likely affects whiich of the following aspects of the host response in this patient?

O A ) Chemotaxis
B ) IgE-mast cell binding
O C ) Leukocyte adhesion
D ) Phagocytosis
E ) Vascular permeability

o
Previous
o
Next
E
(
Lab Values
!
I
Calculator
A
Review
r
Help
&
Pause
Exam Section 2: Item 43 of 50 National Board of Medical Examiners ^ Time Remaining:
Mark Comprehensive Basic Science Self-Assessment 1 hr 13 min 7 sec

43. A previously healthy 43-year -old man comes to the physician because of a 1- week history of fever; vague, migratory abdominal pain ; and generalized muscle pain and weakness . He has no history of a similar illness . His temperature is 38.2°C (100.8DF ) ,
pulse is 73/min respirations are 16/min . and blood pressure is 150/94 mm Hg. Pulmonary examination shows no abnormalities. Abdominal examination shows diffuse tenderness with no organomegaly. There is generalized muscle weakness and tenderness
:

to palpation of various muscle groups. Laboratory studies show :


Hemoglobin 15 g/dL
Hematocrit 45%
Leukocyte count 22,000/mm 3
Platelet count 550 000/mm 3
;

Erythrocyte sedimentation rate 105 mm /h


Urine
Blood 3+
Protein 2+

Blood cultures are negative . Examination of a muscle biopsy specimen from this patient shows segmental transmural necrotizing arteritis . Which of the following is the most likely diagnosis?

£] A ) Churg-Strauss syndrome
o B ) Giant cell arteritis
o C ) Polyarteritis nodosa
o D ) Takayasu arteritis
o E ) Wegener granulomatosis

o
Previous
o
Next
If!
Lab Values
e
Calculator
©
Review
&
Help Pause
Exam Section 2: Item 44 of 50 National Board of Medical Examiners ^ Time Remaining:
Mark Comprehensive Basic Science Self-Assessment 1 hr 13 min 5 sec

44 . A 37 -year -old woman comes to the physician for a follow-up examination 3 months after receiving a kidney transplant from a cadaver donor. She feels well and her condition has been stable since the transplant . She receives standard immunosuppressive
medications Her temperature is 37°C ( 98.6 aF) pulse is 347min respirations are 14/min . and blood pressure is 150/90 mm Hg . Physical examination shows a well-healed surgical scar over the lower abdomen . Serum studies show a creatinine concentration of
?

2 mg/dL ; baseline creatinine concentration was 1.4 mg/dL 1 week post transplant and 2 weeks ago. Serum concentrations of the immunosuppressive drugs are in the low therapeutic range. Examination of a renal biopsy specimen shows lymphocytes in the
tubules and arterial walls. Which of the following best explains these findings ?

O A ) Acute antibody-mediated rejection


o B ) AcuteT lymphocyte mediated rejection
- -

o C ) Adverse effects of immunosuppressant medications


o D ) Chronic allograft rejection
o E ) Hyperacute rejection

o
Previous
o
Next
(£5
Lab Values
e
Calculator
©
Review
&
Help Pause
Exam Section 2: Item 45 of 50 National Board of Medical Examiners^ Time Remaining:
Mark Comprehensive Basic Science Self-Assessment 1 hr 13 min 3 sec

45. A plasmid has acquired a mutation that prevents it from replicating in bacteria . This mutation most likely affects which of the following regions within the plasmid?

0 A ) Antibiotic resistance gene


B ) The tec 2 promoter
0 C ) Multicloning site
D ) Origin of DNAsynthesis
E ) Ribosome binding site
F ) Transposable genetic element

o
Previous
o
Next Lab Values
*
Calculator
m
Review
c*
Help
it*
Pause
Exam Section 2: Item 46 of 50 National Board of Medical Examiners ^ Time Remaining:
Mark Comprehensive Basic Science Self-Assessment 1 hr 13 min 1 sec

46. A 36 -year -old woman comes to the physician for a follow-up examination 1 week after being diagnosed with severe hypertension . Her blood pressure is 130/ 120 mm Hg . Physical examination shows no other
abnormalities. A CT scan of the abdomen shows a renal artery aneurysm . A renal angiogram obtained from a femoral approach is shown . Which of the following is the most likely cause of these findings?

0 A } Atherosclerotic renal artery disease


6 } Calcific nephrosclerosis
C C ) Fibromuscular dysplasia
D ) Polyarteritis nodosa
E } Polycystic kidney disease

o
Previous
o
Next
lE
Lab Values Calculator
m
Review Help Pause
Exam Section 2: Item 47 of 50 National Board of Medical Examiners ^ Time Remaining:
Mark Comprehensive Basic Science Self-Assessment 1 hr 12 min 59 sec

47. A 45 -year -old woman with chronic hepatitis C comes to the physician because of a 3 -week history of fatigue and joint pain . Her temperature is 37.5aC ( 99.5 aF) pulse is 88/min , and blood pressure is 155/75 mm Hg . Physical examination shows the findings in
;

the photograph and hepatosplenomegaly. Serum studies are most likely to show an increased amount of which of the following in this patient?

C '

A ) Cholesterol
B ) Complement
O C ) Creatine kinase
D ) Cryoglobulin
E ) o- Dimer

o
Previous
o
Next
E
Lab Values
*
Calculator
m
Review Help Pause
Exam Section 2: Item 43 of 50 National Board of Medical Examiners ^ Time Remaining:
Mark Comprehensive Basic Science Self-Assessment 1 hr 12 min 56 sec

48. A 39-year -old woman with type 1 diabetes mellitus conies to the physician because of episodes of fainting during the past 3 weeks. The patient' s insulin monitoring has shown that tlhe episodes occur when she becomes hypoglycemic. In the past her
hypoglycemic episodes were characterized by increased sweating , heart rate and shaking of her hands , but now she loses consciousness without warning . Current medications are insulin , lisinopril , and simvastatin . Her pulse is 72/min and blood pressure is
,
:

128/70 mm Hg . Physical examination shows no abnormalities . It is most appropriate for this patient to receive an injection of a drug with which of the following mechanisms of action during these episodes ?

A ) Decreased intestinal absorption of glucose


B ) Enhanced metabolism of iinsulin
C ) Promotion of peripheral insulin uptake
D ) Promotion of skeletal muscle glucose release
E ) Stimulation of hepatic glucose production

Previous Next
It?
Lab Values
*
Calculator
o
Review Help Pause
Exam Section 2: Item 49 of 50 National Board of Medical Examiners ^ Time Remaining:
Mark Comprehensive Basic Science Self-Assessment 1 hr 12 min 54 sec

49. A 34-year -old woman comes to the office because of a 2 -day history of a full-body rash. The rash is not itchy and she has had no other symptoms. She has a staphylococcal abscess on her right buttock for which she has been taking trimethoprim-
;

sulfamethoxazole for the past week . She last had sexual intercourse 2 years ago. She currently is menstruating . She recently adopted a cat and changed laundry detergents to a dye-free variety. She has no history of tick bites. Her temperature is
37.1 X (98.8°F}a pulse is 80/ min , respirations are 16/ min , and blood pressure is 130/85 mm Hg . A photograph of the patient' s right upper extremity is shown : there are similar findings over the trunk and remaining extremities , including the hands and feet . The
mucous membranes appear intact . Examination of the buttocks shows a healed abscess ; the skin has closed , and there is no surrounding cellulitis. Which of the following is the most likely cause of this patient's condition?

A ) Adverse effect of drug


o B ) Cat allergy
o C ) Contact dermatitis
o D ) Tinea corporis
o E ) Toxic shock syndrome

o
Previous
o
Next
It?
Lab Values
*
Calculator
o
Review Help Pause
Exam Section 2: Item SO of 50 National Board of Medical Examiners ^ Time Remaining:
Mark Comprehensive Basic Science Self-Assessment 1 hr 12 min 52 sec

V K

V
• %
\
% I
#•
»

_ *
50. A 30 -year -old man comes to the office because of a 1 -week history of nausea and yellowing of the whites of his eyes. He has no history of serious illness and takes no medications. Vital signs are within normal limits . Abdominal examination shows
hepatomegaly; there is tenderness to palpation over the right upper quadrant Serologic testing is positive for hepatitis A virus IgM antibodies. A high-power photomicrograph of a liver biopsy specimen obtained from a patient with a similar condition is shown .
Which of the following is the most likely mechanism of hepatocyte cell death in this patient?

() A ) Apoptosis caused by activation of the death receptor extrinsic pathway


B ) Apoptosis caused by activation of the mitochondrial intrinsic pathway
O G ) Autophagy caused by inflammation
D ) Necrosis caused by free radical injury
E ) Necrosis caused by lack of blood supply

o
Previous
o
Next Lab Values
*
Calculator
®
Review
f
Help Pause
Exam Section 3: Item 1 of 50 National Board of Medical Examiners^ Time Remaining:
9 Mark Comprehensive Basic Science Self-Assessment 1 hr 14 min 56 sec

1 . In a breast-fed infant, lipase has an integral role in the digestion of which of the following?

(_) A ) Cholesterol
C _ B ) Fatty acids
O C ) Starch
D ) Triglycerides

Next Lab Values Calculator


o
Review Help Pause
Exam Section 3: Item 2 of 50 National Board of Medical Examiners^ Time Remaining:
® Mark Comprehensive Basic Science Self-Assessment 1 hr 14 min 48 sec

2 . Patients with hyperuricemia are most likely to have a deficiency of which of the following enzymes?

0 A ) Adenosine deaminase
B ) Adenine phosphoribosyltransferase
0 C ) Adenylate deaminase
D ) Hypoxanthine -guanine phosphoribosyltransferase
0 E ) Xanthine oxidase

o
Previous
o
Next Lab Values
I!
Calculator
A
Review
m
Help
o
Pause
Exam Section 3: Item 3 of 50 National Board of Medical Examiners ^ Time Remaining:
® Mark Comprehensive Basic Science Self-Assessment 1 hr 14 min 45 sec

3 . A 33 -year-old pregnant woman had genetic amniocentesis that showed a 4S XX karyotype in the cultured amniotic fluid cells . A newborn was delivered with normal male external genitalia . testes and genital ducts . Which of the following is the most likely
; ;

explanation?

O A ) Adrenogenital syndrome
B ) Androgen insensitivity syndrome
O C ) Fragile X syndrome
D ) Maternal exposure to oral progestins
' _ ! E } SRY gene translocation

o
Previous
o
Next Lab Values
I!
Calculator
A
Review
m
Help
o
Pause
Exam Section 3: Item 4 of 50 National Board of Medical Examiners ^ Time Remaining:
® Mark Comprehensive Basic Science Self-Assessment 1 hr 14 min 42 sec

4 . The newborn of a mother with poorly controlled diabetes mellitus has a convulsion and dies shortly after birth. Which of the following morphologic changes is most likely to be seen on autopsy?

0 A ) Basophilic cell hyperplasia of the pituitary gland


B) fi-Cell adenoma of the pancreas
0 C ) Eosinophilic adenoma of the pituitary gland
D ) Hyperplasia of the adrenal zona fasciculata
0 E ) Hyperplasia of the pancreatic islet cells

o
Previous
o
Next Lab Values Calculator
m
Review
r
Help * r*
Pause
Exam Section 3: Item 5 of 50 National Board of Medical Examiners^ Time Remaining:
® Mark Comprehensive Basic Science Self-Assessment 1 hr 14 min 40 sec

5 . A 46 -year-old woman with systemic lupus erythematosus has had chest pain and fever for 2 days. Temperature is 38.3 aC (101°F), pulse is 118/min , and blood pressure is 96/68 mm Hg . Heart sounds are distant . An ECG shows low voltage. Which of the
following types of pulses is most likely in this patient?

O A ) Alternans
B ) Anacrotic
C ) Bigeminal
D ) Bisferiens
E ) Paradoxus

o
Previous
o
Next
(E
Lab Values
I!
Calculator
A
Review
&
Help
tl*
Pause
Exam Section 3: Item 6 of 50 National Board of Medical Examiners ^ Time Remaining:
Mark Comprehensive Basic Science Self-Assessment 1 hr 14 min 37 sec

6 . A 27 -year-old man has the sudden onset of fever and shaking chills 2 weeks after returning from a trip fo Africa . He has mild anemia . A peripheral blood smear is shown . He is treated with chloroquine followed by
primaquine. Which of the following best explains the benefit of using primaquine in this patient?

0 A ) Decreases the dose of chloroquine needed


6 } Decreases the duration of chloroquine needed
C C ) Produces more rapid resolution of symptoms : i
11
* ’

D ) Treats chloroquine - resistant parasites


JJ

E } Treats exoerythrocytic parasites

O
Previous
O
Next Lab Values ^
Calculator
A
Review Help Pause
Exam Section 3: Item 7 of 50 National Board of Medical Examiners^ Time Remaining:
® Mark Comprehensive Basic Science Self-Assessment 1 hr 14 min 34 sec

7 . A 3G -year-old man who is seropositive for HIV develops nausea : vomiting , watery diarrhea without blood abdominal cramps: and a 2.3 -kg ( 5-lb ) weight loss over 10 days . A diagnosis is made by microscopic examination of stool . Which of the following is the
;

most likely causal organism?

O A ) Clostridium difficile
B } Cryptosporidium species
O C ) Enterococcus species
D ) Escherichia coli 0157 : H7
E ) Helicobacter pylon
F ) Mycobacterium avium-intracellulare
O G } Pseudomonas species
H ) Shigella dysenteriae
CJ I ) Toxoplasma gondii
IJ

J } Vibrio choleme

o
Previous
o
Next
E
(
Lab Values
*
Calculator
m
Review Help Pause
Exam Section 3: Item 8 of 50 National Board of Medical Examiners ^ Time Remaining:
Comprehensive Basic Science Self-Assessment 1 hr 14 min 32 sec

8 . A 25 -year-old woman gravida 2 , para 2 comes to the physician because of a painful lump in the right vulva . Her last menstrual period was 1 week ago. She does not recall any recent trauma and has had no prior vaginal infections. Examination shows an
;

acutely tender. 2.5 -cm mass that is located near the vaginal orifice posteriorly and distends the labia majora . It appears to point vaginally. Which of the following is the most likely diagnosis ?

O A ) Bartholin abscess
B ) Inclusion cyst
O C ) Skene gland occlusion
D ) Vulvar carcinoma
E ) Vulvar hematoma

o
Previous
o
Next Lab Values
*
Calculator
m
Review Help Pause
Exam Section 3: Item 9 of 50 National Board of Medical Examiners ^ Time Remaining:
Mark Comprehensive Basic Science Self-Assessment 1 hr 14 min 30 sec

9 . A 63 -year-old man with Parkinson disease is being treated with levodopa and carbidopa . He is doing well and has few adverse effects. Concurrent therapy with levodopa and carbidopa is useful because of which of the following characteristics of carbidopa ?

0 A ) Blocks muscarinic receptors


B ) Does not cross the blood - brain barrier
0 C ) Enhances activity of the transporter for levodopa
D ) Increases intestinal metabolism of levodopa
0 E ) Inhibits monoamine oxidase

o
Previous
o
Next Lab Values Calculator
m
Review Help Pause
Exam Section 3: Item 10 of 50 National Board of Medical Examiners ^ Time Remaining:
Mark Comprehensive Basic Science Self-Assessment 1 hr 14 min 13 sec

c Normal
10. A 7-year-old girl is admitted to the hospital because of severe anemia . Hemolytic anemia with a probable defect in glycolytic energy production is diagnosed . Results of pyruvate kinase activity in erythrocyte E*
lysates using varying concentrations of phosphoenolpyruvate are shown . Which of the following best explains these findings? Patient
E
o
0 A ) Decrease in the rate of enzyme synthesis E

6 } Increase in the rate of enzyme degradation &


o

o C ) Mutation in the enzyme that decreases V


0

max >
' '

_ D ) Mutation in the enzyme that increases Km [Phosphoenolpyrijvatej

E ) Presence of a noncompetitive inhibitor in cells


F ) Presence of an irreversible inhibitor in cells

Previous
o o
Next
(E
Lab Values
!
I
Calculator
A
Review
m
Help
o
Pause
Exam Section 3: Item 11 of 50 National Board of Medical Examiners ^ Time Remaining:
Mark Comprehensive Basic Science Self-Assessment 1 hr 14 min 11 sec

11 . An experiment is designed to assay the secretions of chondrocytes obtained from the articular cartilage of osteoarthritic joints. Which of the following compounds is most likely to be found in decreased concentration in the secretions ?

0 A ) Catabolic metalloproteinase
B ) lnterleukin-1 ( IL- 1 )
0 C ) InterleuKcin -G (IL-6)
D ) Proteoglycans
0 E } Tumor necrosis factor

o
Previous
o
Next Lab Values Calculator
m
Review Help Pause
Exam Section 3: Item 12 of 50 National Board of Medical Examiners ^ Time Remaining:
Comprehensive Basic Science Self-Assessment 1 hr 14 min 9 sec

12 .
Time Serum Osmolality Urine Osmolality Urine Flow Rate
( minutes) ( mOsmol/kg) ( mOsmol/ kg ) (mL / min)
0 290 305 3.0
30 285 410 2.0
60 280 650 1.5
90 275 710 1.0

The table shows serum osmolality, urine osmolality, and urine flow rate measured at 30-minute intervals in a 30 -year -old man . Which of the following substances was most likely administered at time 0?

I A ) ADH (vasopressin )
C_ B ) Aldosterone
O C ) Atrial natriuretic peptide
D ) Furosemide
E ) Mannitol

o
Previous
o
Next
tS
Lab Values
4=
Calculator
m
Review Help Pause
Exam Section 3: Item 13 of 50 National Board of Medical Examiners ^ Time Remaining:
Comprehensive Basic Science Self-Assessment 1 hr 14 min 7 sec

2500 -
13. A healthy 40 -year -old man is participating in a study in which he is given an intravenous injection of insulin to induce hypoglycemia. The graph shows serum concentrations of catecholamines at baseline and the
E
peak values at the nadir of serum glucose . Which of the following most likely underlies the different magnitude of the epinephrine and norepinephrine responses? iTT.i
a 2000 -
CD
0 A } Activation of catechol - O-methyltransferase £
=
6 } Failure of monoamine oxidase to metabolize epinephrine I
o
1500 -

o
C C ) Increased stability of circulating epinephrine B
03
O
1000 -
D ) Reuptake of norepinephrine into sympathetic synapses 03

I 500 -
E } Selective activation of medullary epinephrine release _
0
Epinephrine Norepinephrine

Baseline
Post - insulin

O
Previous
O
Next Lab Values
*
Calculator Review Help Pause
Exam Section 3: Item 14 of 50 National Board of Medical Examiners ^ Time Remaining:
0 Mark Comprehensive Basic Science Self-Assessment 1 hr 14 min 4 sec

14 . An analysis of patients admitted to a specific hospital finds that the risk for myocardial infarction is higher in men with chronic obstructive pulmonary disease ( CORD) than in men without CORD ( odds ratio 5.4 , 95% confidence interval: 1.4-10.1). These results
can most reasonably be generalized to which of the following populations ?

O A ) Men with CORD and myocardial infarction


B ) Men with CORD in the hospital
O C ) Men with COPD living at home
D ) Men with myocardial infarction in the hospital
E ) All patients with COPD and myocardial infarction
F ) All patients with COPD in the hospital
O G ) All patients with COPD living at home
H ) All patients with myocardial infarction

4= m
Previous Next Lab Values Calculator Review Help Pause
Exam Section 3: Item 15 of 50 National Board of Medical Examiners ^ Time Remaining:
0 Mark Comprehensive Basic Science Self-Assessment 1 hr 14 min 1 sec

15. The following data about cancer prevalence and mortality are collected in a small city for a calendar year :
Males Females Total
Population 43,000 47: G 00 90 ,000
Number of persons with cancer ( all types ) 224 183 407
Number of deaths from cancer ( all types ) 50 30 80
Number of women with ovarian cancer 8 3
Number of deaths from ovarian cancer 5 5

Which of the following is the period prevalence of ovarian cancer in women in this city in this calendar year?

G A ) 5/183
O B ) 5/ 407
O C ) 5/47,000
C D ) 5/ 90,000
O E ) 8/183
O F } 8/407
O G ) 8/47,000
C H ) 8/ 90,000

o
Previous
o
Next
tS
Lab Values
4=
Calculator
m
Review Help Pause
Exam Section 3: Item 16 of 50 National Board of Medical Examiners ^ Time Remaining:
Mark Comprehensive Basic Science Self-Assessment 1 hr 13 min 58 sec

16. A 34-year-old man is brought to the emergency department semiconscious and combative. In addition to sedation , a short-acting neuromuscular blocking agent is administered for intubation to prevent aspiration. Within a few seconds after administration of
the drug he has transient muscle fasciculations in his face : he develops generalized paralysis within 1 minute. Forty-five minutes after completion of the procedure , he is still paralyzed . A genetic abnormality of which of the following enzymes is the most likely
,

cause of his unusually slow recovery from paralysis ?

A ) Angiotensin-converting enzyme
B ) Choline O- acetyltransferase
C ) Monoamine oxidase
D ) Ptienylethanolamine A/-methyltransferase
E ) Pseudocholinesterase
F ) Tyrosine hydroxylase

o
Previous
o
Next
It?
Lab Values
*
Calculator
o
Review Help Pause
Exam Section 3: Item 17 of 50 National Board of Medical Examiners ^ Time Remaining:
Mark Comprehensive Basic Science Self-Assessment 1 hr 13 min 53 sec

17. A 42 -year-old man conies to the physician because of progressive weakness of his hands during the past 10 months. Physical examination shows atrophy of the hand muscles with fasciculations. There are also fasciculations of the tongue and trapezius
muscles. Electromyography confirms the diagnosis of amyotrophic lateral sclerosis. Which of the following initial statements by the physician is most appropriate ?

O A ) "You have amyotrophic lateral sclerosis , also called Lou Gehrig disease , which is a terminal illness. You will eventually need to go on a respirator. Have you thought about advance directives ?"
B ) "You have an incurable illness. I' m sorry to say there is nothing I can do for you . Are you religious ?"
O C ) ''You have Lou Gehrig disease. Have you ever heard anything about this disease before ? 11

D ) '''You have a mild case of amyotrophic lateral sclerosis , also called Lou Gehrig disease. We ' ll check it every few months just to see if it is progressing ."
! E ) "You may have amyotrophic lateral sclerosis. We’ll check every few months and if it progresses, then we’ll know for sure. "

o
Previous
o
Next Lab Values
*
Calculator
®
Review
r
Help
r
Pause
Exam Section 3: Item 13 of 50 National Board of Medical Examiners ^ Time Remaining:
Mark Comprehensive Basic Science Self-Assessment 1 hr 13 min 51 sec

18. A 50 -year-old woman comes to the physician because of progressive shortness of breath during the past 2 years . Her respirations are 20/min . Physical examination shows cyanosis and ankle edema . Her pulmonary artery pressure pulmonary vascular
,

<
resistance , and right atrial pressure are increased ; her pulmonary capillary wedge pressure is 9.3 mm Hg N= S— 16 ) . Her ventilation improves when inhaled nitric oxide is administered. Treatment with oral bosentan is begun for long-term therapy. Which of the
following mediators of pulmonary resistance in this patient will most likely be antagonized by this drug?

A ) Adenosine
B ) Calcitonin gene-related peptide
C ) C -reactive protein
D ) Endothelin
C_ E } P rostacycliin (PGI 2)

o
Previous
o
Next
(£5
Lab Values
e
Calculator
&
Review
ft
Help
f*
Pause
Exam Section 3: Item 19 of 50 National Board of Medical Examiners ^ Time Remaining:
Mark Comprehensive Basic Science Self-Assessment 1 hr 13 min 47 sec

19. A 63 -year-old man conies to the emergency department in a mountain resort town with his wife because of a 1 - month history of shortness of breath and headaches. One month earlier he retired and moved from his home at sea level to the resort town which
is 9000 feet above sea level . His wife is asymptomatic . His respirations are 16/niin Laboratory studies show an erythrocyte count of 4 million/mm 3 This patient most likely has a history of disease of which of the following organs ?

C _) A ) Heart
B ) Kidney
O c ) Liver
D ) Lung
E ) Thyroid gland

o
Previous
o
Next
E
(
Lab Values
I!
Calculator
A
Review
&
Help
tl*
Pause
Exam Section 3: Item 20 of 50 National Board of Medical Examiners^ Time Remaining:
® Mark Comprehensive Basic Science Self-Assessment 1 hr 13 min 44 sec

20. In a clinical study of a molecular variant hypothesized to be related to schizophrenia 200 subjects are examined for a particular single nucleotide polymorphism. The results are shown in the table.
;

Genotype Number of Controls


AA 50
AG SO
GG 70

If A is one allele and G is the other, which of the following is the frequency of allele A?

O A ) 130/200
B ) 130/400
O C ) 140/200
C D ) 140/400
O E ) 180/200
C F } 180/400

o
Previous
o
Next Lab Values Calculator
m
Review Help Pause
Exam Section 3: Item 21 of 50 National Board of Medical Examiners^ Time Remaining:
Comprehensive Basic Science Self-Assessment 1 hr 13 min 41 sec

21 . A 69 -year -old woman comes to the physician because she has had weakness of her left leg since awakening that morning . Physical examination shows weakness of the extremity. A Babinski sign is present on the left . Sensory testing shows decreased
somatic sensation in the left foot agraphesthesia on the plantar surfaces of the toes and decreased position sense in the toes . An MRI of the brain shows an edematous area in the cerebral cortex of the right hemisphere . The most likely cause of this
,
;

condition is a lesion located at which of the following labeled areas in the photographs of normal brains shown?

- \

- /
A)
- B)
; c
:- )
\

D
- \

E)
- F)
-
; G)
/

U H)
\ J

:: J)

Previous Next Lab Values Calculator Review Help Pause


Exam Section 3: Item 22 of 50 National Board of Medical Examiners ^ Time Remaining:
Mark Comprehensive Basic Science Self-Assessment 1 hr 13 min 37 sec

22 . A 43-year -old man with a 10-year history of alcoholism comes to the physician because of a change in skin color. The patient says that he has been taking two extra- strength acetaminophen tablets every 4 to 6 hours for the past 3 days for a severe headache.
Physical examination shows jaundice. Laboratory studies show an increased prothrombin time and a markedly increased serum AST activity. Alteration in which of the following metabolites within hepatocytes is most likely associated with his illness ?

O A ) Decreased glucuronide conjugates


B ) Decreased glutathione
O C } Decreased NAD +

D ) Decreased NADH
E ) Increased glucuronide conjugates
F ) Increased glutathione
O G) Increased NAD +

CJ H ) Increased NADH

(£5 e © &
Previous Next Lab Values Calculator Review Help Pause
Exam Section 3: Item 23 of 50 National Board of Medical Examiners ^ Time Remaining:
Mark Comprehensive Basic Science Self-Assessment 1 hr 13 min 33 sec

23. A 55 -year -old woman is brought to the emergency department by police 1 hour after she was found wandering around the city park at midnight , appearing confused . She told the police that she is a prominent physician and a member of the city council . She is
unwashed,, malodorous. and disheveled . She is 160 cm (5 ft 3 in) tall and weighs 55 kg (121 lb): BMI is 21 kg /m 2 Her pulse is 82/min , and blood pressure is 110/82 mm Hg . Physical examination shows a broad-based gait and nystagmus . She is not oriented
to person, place, or time, but she is pleasant and cooperative. Results of alcohol and drug screening are negative. An MRI of the brain is most likely to show atrophy of which of the following?

A ) Amygdala
B ) Hippocampus
C ) Hypothalamus
D ) Mammillary bodies
C_ E ) Parietal lobe

o
Previous
o
Next Lab Values
*
Calculator
©
Review
&
Help Pause
Exam Section 3: Item 24 of 50 National Board of Medical Examiners ^ Time Remaining:
Mark Comprehensive Basic Science Self-Assessment 1 hr 13 min 31 sec

24 . A healthy 27-year -old woman consumes a. turkey sandwich : a cup of sweetened yogurt an apple and three cookies for lurch . After lunch: she returns to working at a computer. Two hours later she drinks a 16 -ounce bottle of cola sweetened with high
,
:

fructose corn syrup and sucrose. The activity of which of the following enzymes is most likely increased in this patient’s liver?

O A ) Carnitine palmitoyltransferase I
B ) Fatty acid synthase
O C ) Glycogen phosphorylase
D ) Phosphoenolpyruvate carboxykinase
E ) Protein kinase A

4= m c* it*
Previous Next Lab Values Calculator Review Help Pause
Exam Section 3: Item 25 of 50 National Board of Medical Examiners ^ Time Remaining:
Mark Comprehensive Basic Science Self-Assessment 1 hr 13 min 29 sec

25. During a study of familial hypercholesterolemia hepatocytes from an affected patient are extracted and analyzed . Results show that the LDL receptors are evenly distributed on the surface of the cells. The cells also bind LDL-cholesterol with normal affinity
,

but cannot internalize it . The cells are shown to internalize transferrin . Based on these findings, which of the following is the most likely location of the causal mutation in this patient?

O A) ApoB
0 B) ApoE
O C ) Clathrin heavy chain
D ) Clathrin light chain
E ) Cytoplasmic domain of the LDL receptor
;_ F ) Extracellular domain of the LDL receptor

o
Previous
o
Next
lE
Lab Values Calculator
m
Review Help Pause
Exam Section 3: Item 26 of 50 National Board of Medical Examiners ^ Time Remaining:
Mark Comprehensive Basic Science Self-Assessment 1 hr 13 min 23 sec

26. A 23 -year -old man conies to the physician because of a 2-week history of light-headedness, mild chest pain : palpitations and shortness of breath after exercising . He has no history of major medical illnesses . His temperature is 36 . ETC {98.4DF ) pulse is
; :

72/ min respirations are 17/min . and blood pressure is 120/82 mm Hg . Cardiac examination shows a bounding point of maximal impulse that is shifted to the left. Echocardiography shows cardiomegaly and marked thickening of the ventricular septum. It is
;

most appropriate for the initial therapy to increase which of the following in this patient?

A ) Contractility
B ) Diastolic filling
C ) Heart rate
D ) Peripheral resistance
E ) Pulmonary perfusion pressure

m
Previous Next Lab Values Calculator Review Help Pause
Exam Section 3: Item 27 of 50 National Board of Medical Examiners ^ Time Remaining:
Mark Comprehensive Basic Science Self-Assessment 1 hr 13 min 23 sec

27. An 18-year - old man with Crohn disease is admitted to the hospital because of a 1 - day history of severe abdominal pain and intermittent bloody diarrhea . His temperature is 38 X {100 4DF), pulse is 9 S/min . and respirations are 18/min . Physical examination
shows a draining anal fistula . Treatment with broad - spectrum antibiotics and prednisone, along with intravenous hydration and parenteral nutrition . is initiated . He recovers over the next 3 weeks. In addition to resolving the infection , the most likely mechanism
of action of this pharmacotherapy is suppression of which of the following?

A ) Antibody binding
B ) Complement activity
C ) Mast cell degranulation
D ) Neutrophil function
! _ E } T- lymphocyte function

o
Previous
o
Next
E
(
Lab Values
I!
Calculator
A
Review Help Pause
Exam Section 3: Item 23 of 50 National Board of Medical Examiners ^ Time Remaining:
Mark Comprehensive Basic Science Self-Assessment 1 hr 13 min 20 sec

28. A previously healthy 18-year -old woman conies to the physician because of a 3- day history of pain with urination and urinary frequency and urgency. Physical examination shows mild tenderness over the suprapubic region . A urine culture grows an organism
that is lactose fermenting and spot-indole test positive. The organism is resistant to ampicillin but sensitive to ceftriaxone . Which of the following is the most likely mechanism of antimicrobial resistance exhibited by this organism?

O A ) Alteration of the existing penicillin -binding protein


B ) Changes iin porin
C ) Efflux pumps
D ) Elaboration of a new penicillin -binding protein
E ) p - Lactamase production

o
Previous
o
Next Lab Values Calculator
m
Review
c*
Help
it*
Pause
Exam Section 3: Item 29 of 50 National Board of Medical Examiners ^ Time Remaining:
Mark Comprehensive Basic Science Self-Assessment 1 hr 13 min IT sec

29. A 40 -year -old woman comes to the physician because of a 3 -day history of pain and swelling of her left leg. One week ago . she underwent operative removal of several sebaceous cysts over the labia majora. Physical examination shows edema of the left
,

lower extremity and purulent material draining from one of the operative incisions . This patient most likely has an infection that spread to which of the following groups of lymph nodes ?

O A ) External iliac
B ) Inguinal
O C ) Internal iliac
D ) Obturator
0 E ) Sacral

o
Previous
o
Next
E
(
Lab Values
!
I
Calculator
A
Review
m
Help
o
Pause
Exam Section 3: Item 30 of 50 National Board of Medical Examiners ^ Time Remaining:
Mark Comprehensive Basic Science Self-Assessment 1 hr 13 min 14 sec

30. A 57 -year -old woman comes to the physician 3 days after she found a lump in her left breast on self-examination . Physical examination shows a 1 3-cm mass in the left breast. Microscopic examination of a biopsy specimen of the mass shows carcinoma . 11 is
,

most appropriate for the physician to consider which of the following when grading the carcinoma ?

O A ) Distant metastasis
B ) Maximum tumor size
O C ) Number of mitoses per unit area
D ) Peritumoral lymphocytic response
E ) Presence of extracellular mucin
F ) Regional lymph node metastasis

o
Previous
o
Next Lab Values
I!
Calculator
A
Review
&
Help
tl*
Pause
Exam Section 3: Item 31 of 50 National Board of Medical Examiners ^ Time Remaining:
Mark Comprehensive Basic Science Self-Assessment 1 hr 13 min 11 sec

31. A 35 -year -old man is brought to the physician by staff of the group home where he resides because of worsening behavior for 2 weeks. During the interview he tells the physician that he believes the CIA is spying on him through his television set . He then
;

reports hearing voices in the hall outside the examination room and states that the CIA now plans to kill him . He appears disheveled with unkempt hair and poor hygiene. He has difficulty answering the physician' s questions because he seems to be listening
to internal stimuli . Mental status examination of the patient is most likely to show which of the following findings ?

A ) Flattened affect
B ) Inability to state his name
C ) Inability to write his name
D ) Lack of orientation to place or time
E ) Long-term memory impairment

o
Previous
o
Next Lab Values Calculator
m
Review Help Pause
Exam Section 3: Item 32 of 50 National Board of Medical Examiners ^ Time Remaining:
0 Mark Comprehensive Basic Science Self-Assessment 1 hr 13 min 9 sec

32 . A 12 -year -old boy is brought to the physician by his mother because of a persistent rash around his nose and mouth for 4 months. He also has had a decreased appetite and an unintentional 4.5 - kg (10- lb) weight loss during the past 3 months. His mother
states, "The rash developed shortly after he started at his new school . I wonder if he keeps getting into something . I 've tried hydrocortisone cream but it didn't work." She adds, "And now his grades are falling , and he’s not eating as much as he used to . I
,

hope he ' s not depressed because of the rash." The rash began as mild redness and then became dry and started to flake . The patient says there is no pain or discomfort with the rash . Physical examination shows injection of both conjunctivae, an
eczematous eruption extending interiorly from the nasolabial fold to the upper lip, and an erythematous nasal mucosa that is friable with gold stippling .. When asked he states that his mood is fine . Which of the fallowing is the most likely cause of this patient' s
,

condition?

0 A ) Atopic dermatitis
B ) Impetigo
0 C ) Inhalant abuse
D ) Mercury poisoning
E ) Wegener granulomatosis

o
Previous
o
Next
(£5
Lab Values
e
Calculator
©
Review
&
Help Pause
Exam Section 3: Item 33 of 50 National Board of Medical Examiners ^ Time Remaining:
0 Mark Comprehensive Basic Science Self-Assessment 1 hr 13 min 5 sec

33. A male newborn is delivered at term with a right- sided aortic arch . Physical examination shows full cheeks , low-set ears: and a small chin . Serum studies show a decreased calcium concentration . Which of the fallowing is the most likely site of the
malformations in this patient?

O A ) First and second branchial arches


B ) First and second branchial grooves
O C ) First and second branchial pouches
D ) Third and fourth branchial arches
E ) Third and fourth branchial grooves
F ) Third and fourth branchial pouches

m
Previous Next Lab Values Calculator Review Help Pause
Exam Section 3: Item 34 of 50 National Board of Medical Examiners ^ Time Remaining:
0 Mark Comprehensive Basic Science Self-Assessment 1 hr 13 min 2 sec

34 . A 75 -year -old man is found to have an 8 -cm mass in the head of the pancreas that is encasing and markedly narrowing the inferior vena cava . Blood from this patient' s lower extremities is most likely returning to the superior vena cava from which of the
following veins?

O A ) Celiac and paraspinal veins


B ) Lumbar, azygos and hemiazygos veins
;

O C ) Right and left renal veins and suprarenal veins


D ) Superior and inferior mesenteric veins
' _ ! E } Superior mesenteric and hepatic veins

o
Previous
o
Next Lab Values Calculator
m
Review Help Pause
Exam Section 3: Item 35 of 50 National Board of Medical Examiners ^ Time Remaining:
Mark Comprehensive Basic Science Self-Assessment 1 hr 12 min 59 sec

35. A 40 -year -old woman comes to the physician because of pain of her hands wrists and knees for 2 weeks . Physical examination shows swelling tenderness and warmth of the proximal interphalangeal , wrist , and knee joints. Laboratory studies show an
; : ; ;

increased erythrocyte sedimentation rate and normochromic normocytic anemia . A test result for rheumatoid factor is positive. Which of the following sets of findings in synovial fluid obtained from the knee joint is most likely in this patient?

Complement ( C 3 ) Segmented Neutrophils lnterleukin- 1 (IL- 1 ) Tumor Necrosis Factor


O A)
O B) .
..

O C}
O D) .
..

E) , ,

0 F) i i i

o
Previous
o
Next Lab Values Calculator
0
Review
9
Help
0
Pause
Exam Section 3: Item 36 of 50 National Board of Medical Examiners ^ Time Remaining:
Mark Comprehensive Basic Science Self-Assessment 1 hr 12 min 56 sec

Sv - +m I V"

**
ft

36. A 23-year -old woman at 32 weeks' gestation comes to the emergency department because of a 1 -day history of left flank pain and fever. Her temperature is 39.1 DC (102.3T ), pulse is 104/ min respirations are 14/min , v <p * D eW *
; f
?. 4*
I. m wP M Hi
* ft

* A* te
*
* K

and blood pressure is 120/ 72. mm Hg . Physical examination shows prominent tenderness over the left costovertebral angle. A photomicrograph of a renal biopsy specimen from a similar patient is shown . Which of the i • §
i i |]
i

V tVh I
r -
following is the most likely diagnosis? * - i
*
4 fc
lift
f
- *
Mf
r
- f ^V ^
1

*
F

*
C A } Acute pyelonephritis 4
m
J
- *
*
*
*
-- fci

h
a
-^ .+ »

r
Mr.
_-
I

> v
V '
-JF" 43 F

6 } Acute renal infarction '


4

- .®
f

- i fc
S
- 4
'
C C ) Acute tubulointerstitial nephritis "L
vvV ?*Vv> fj I
aTO '
1

D ) Crescentic glomerulonephritis
L
*5*
ygM
'

-
i
A
4
- #

f Si
E ) Hemolytic uremic syndrome Vi
i
*4 . I
mM
*r 5
*
5 *
l * T8

* £ SJf i

sW > p
i1!

4,1

o
Previous
o
Next Lab Values Calculator
<r
Review Help Pause
Exam Section 3: Item 37 of 50 National Board of Medical Examiners ^ Time Remaining:
Comprehensive Basic Science Self-Assessment 1 hr 12 min 53 sec

37. A SS -year -old woman is brought to the emergency department because of progressive shortness of breath during the past month . Two weeks ago, she underwent coronary artery bypass grafting . Her respirations are 26/min. There is dullness to percussion at
the left base of the chest . A chest x-ray shows substantial fluid in the left pleural cavity. Thoracentesis yields 1200 mL of pale, milky fluid that is identified as chyle. Which of the following procedures during the operation most likely produced the iatrogenic
chylothorax?

A ) Dissection of the internal thoracic artery


B ) Midsternal thoracotomy
C ) Placement of the central line via the left internal jugular vein
D ) Placement of the epicardial pacemaker lead
! _ E ) Placement of the pulmonary artery catheter via the right subclavian vein

o
Previous
o
Next
It?
Lab Values
*
Calculator
0
Review
9
Help
0
Pause
Exam Section 3: Item 33 of 50 National Board of Medical Examiners ^ Time Remaining:
Mark Comprehensive Basic Science Self-Assessment 1 hr 12 min 50 sec

38. A 27-year -old woman comes to the physician 3 weeks after the onset of blurred vision in her right eye that has been improving during the past 5 days. She had a similar episode 1 year ago while working
on a ranch in northeastern Arizona : the episode resolved in 2 weeks without treatment . She has no history of major medical illness. Ophthalmologic examination shows a right afferent pupillary defect.
Visual acuity is 20/ 20 in the left eye and 20/200 in the right eye . Neurologic examination shows mild weakness of the lower two thirds of the right side of the face and a right pronator drift . There is mild
dysmetriia when she moves the left extremities. An MRI of the brain is shown : the arrows indicate multiple abnormalities. Which of the following is the most likely diagnosis ?

C A ) Cerebral abscesses
6 } Cerebral lymphoma
O C ) Coccidioidomycosis
D ) Multiple sclerosis
E ) Neurocysticercosis

Previous Next Lab Values


*
Calculator
®
Review Help
^
Pause
Exam Section 3: Item 39 of 50 National Board of Medical Examiners ^ Time Remaining:
Comprehensive Basic Science Self-Assessment 1 hr 12 min 34 sec

39. A 2 -year-old boy is brought to the physician because of a 1 -week history of high -grade fever and rash . His temperature is 39.6DC ( 1G 3.3°F), pulse is 144/ min : and blood pressure is 122/76 mm Hg . Physical examination shows marked conjunctival injection and
erythema and edema of the hands and feet . A photograph of the cheeks and mouth is shown . Echocardiography shows dilation of the proximal coronary arteries . Which of the following is the most likely diagnosis ?

G A } Acute rheumatic fever


B ) Ehrlichiosis
O C) Erythema multiforme
D ) Mucocutaneous lymph node syndrome ( Kawasaki disease)
G E ) Scarlet fever

o
Previous
r
Next Lab Values
<=
Calculator
m
Review Help Pause
Exam Section 3: Item 40 of 50 National Board of Medical Examiners ^ Time Remaining:
Mark Comprehensive Basic Science Self-Assessment 1 hr 12 min 31 sec

40. A 65 -year -old woman comes to the emergency department because of a 4 -hour history of vomiting bright red blood : she also has had dizziness and nausea during this period. Her pulse is 140/ mir and blood pressure is 70/ 30 mm Hg . Physical examination
;

shows pale clammy skin diaphoresis, and decreased capillary refill time . A diagnosis of hypovolemic shock is made. Placement of a catheter in the internal jugular vein for blood transfusions is planned . Improper insertion of the catheter is most likely to result
; :

in damage to which of the following sets of underlying structures ?

A ) Common carotid artery and pulmonary artery


B ) Esophagus and common carotid artery
C ) Esophagus and pulmonary artery
D ) Lung and common carotid artery
E ) Lung and esophagus

o
Previous
o
Next
It?
Lab Values
*
Calculator
®
Review
f
Help Pause
Exam Section 3: Item 41 of 50 National Board of Medical Examiners ^ Time Remaining:
Mark Comprehensive Basic Science Self-Assessment 1 hr 12 min 29 sec

41. A 57-year -old woman comes to the emergency department because of a 7-week history of fever night sweats, and nonproductive cough; she also has had a 6.8-kg (15 -lb) weight loss during this period .
She is currently receiving pharmacotherapy for Crohn disease but does not remember the name of the medication . Her temperature is 38.2 X (100 8°F) pulse is 9 Q /min respirations are 23/miin : and
; ;

blood pressure is 130/00 mm Hg . Amphoric breath sounds are heard over the right upper lung field . A chest x - ray is shown . This patient is most likely being treated with a monoclonal antibody targeted
against which of the following?

C A ) CD 3
O B ) CD20
O C ) Complement protein C5
O D ) IgE
£ E ) Tumor necrosis factor- a

o
Previous
o
Next Lab Values
*
Calculator
©
Review
&
Help Pause
Exam Section 3: Item 42 of 50 National Board of Medical Examiners ^ Time Remaining:
Mark Comprehensive Basic Science Self-Assessment 1 hr 12 min 24 sec

42 . A 22 -year -old woman comes to the physician for a follow-up examination . One year ago she was diagnosed with a pulmonary embolism . Two years ago, she delivered a female stillborn at 23 weeks' gestation. Physical examination today shows no
:

abnormalities . Laboratory studies show a platelet count of 250,00 G/mrn3, a prothrombin time within the reference range : and an increased part al thromboplastin time . The findings in this patient are most consistent with which of the following conditions ?

O A ) Antiphospholipid antibody syndrome


B ) Factor V Leiden mutation
O C ) Increased factor VIII (antihemophilic factor) concentration
D ) Protein C deficiency
CJ E ) Prothrombin G2021OA mutation

o
Previous
o
Next
E
(
Lab Values
I!
Calculator
A
Review Help Pause
Exam Section 3: Item 43 of 50 National Board of Medical Examiners ^ Time Remaining:
Mark Comprehensive Basic Science Self-Assessment 1 hr 12 min 21 sec

43. A 16-year-old boy is brought to the emergency department because of a 3-day history of episodes of sharp chest pain on the left that lasts for a few seconds. When the patient is asked to indicate the location of the pain he points to the 4th rib on the left. The
,

pain is not associated with change in position and it does not radiate. There is no history of trauma . The patient appears anxious . His pulse is 90/min and regular . The left anteromedial area of the 4th rib is tender to palpation . Cardiac and pulmonary
examinations show no abnormalities . Which of the following is the most likely diagnosis ?

A ) Anxiety
B ) Costochondritis
C ) Pericarditis
D ) Pneumonia
C_ E ) Rib fracture

o
Previous
o
Next Lab Values Calculator
m
Review
c*
Help
it*
Pause
Exam Section 3: Item 44 of 50 National Board of Medical Examiners ^ Time Remaining:
Mark Comprehensive Basic Science Self-Assessment 1 hr 12 min IS sec

44 . A married couple and their 12 -year- old son come to the emergency department because of a 2 -hour history of nausea , vomiting , and diarrhea . They all ate lunch at a company picnic 6 hours ago. The physician suspects staphylococcal food poisoning and
notifies the local health department . The health department contacts other picnic attendees to collect data on which foods they consumed at the event . Based on the investigation , ice cream is suspected as the source of the toxin. Among those who ate the ice
cream 96 became ill and 24 did not experience symptoms. Among those who did not eat the ice cream : 3 became ill and 27 did not experience symptoms. Which of the following is the relative risk for illness in those who ate the ice cream compared with
those who did not?

O A ) 0.125
G B ) 0.7
o C) 8
o D ) 32
o E ) 36

o
Previous
o
Next Lab Values
*
Calculator
O
Review Help
tt*
Pause
Exam Section 3: Item 45 of 50 National Board of Medical Examiners^ Time Remaining:
Mark Comprehensive Basic Science Self-Assessment 1 hr 12 min 15 sec

45. A 53 -year -old man conies to the physician because of a 2-week history of severe chest pain when climbing stairs . He does not have chest pain at rest . The diagnosis of angina pectoris is made. Nitroglycerin is prescribed for this patient because of which of
the following mechanisms of action?

O A ) Decreases cAMP
B ) Decreases cGMP
O C ) Decreases inositol 1 : 4.5-trisphosphate
D ) Increases cAMP
E ) Increases cGMP
F ) Increases inositol 1 : 4.5 -trisphosphate

o
Previous
o
Next Lab Values Calculator
m
Review Help Pause
Exam Section 3: Item 46 of 50 National Board of Medical Examiners ^ Time Remaining:
Mark Comprehensive Basic Science Self-Assessment 1 hr 12 min 13 sec

46. A 45 -year -old man conies to the physician because of a 2-week history of pain of his left testis . Six weeks ago he had a vasectomy. Physical examination shows a bead-like mass of the left testis . Excision of the mass is done. Microscopic examination of the
;

mass shows sheets of extravasated spermatozoa engulfed by histiocytes, many with prominent enlarged nuclei . This patient' s lesion is most likely the result of which of the following processes?

C _) A ) Adaptation
B ) Autoimmunity
O C ) Hemorrhage
D ) Infection
E ) Neoplasia

o
Previous
o
Next Lab Values Calculator
m
Review
c*
Help
it*
Pause
Exam Section 3: Item 47 of 50 National Board of Medical Examiners ^ Time Remaining:
Mark Comprehensive Basic Science Self-Assessment 1 hr 12 min 11 sec

47. A 2 -year-old boy is brought to the physician because of chronic bacterial respiratory infections since birth . He is currently asymptomatic . Physical examination shows no abnormalities. T- and B -lyimphocyte counts and serum antibody concentrations are within
the reference ranges. Natural killer cell count and function are normal . Analysis of cellular expression of human leukocyte antigen by flow cytometry shows absence of class I MHC -expressing cells . A diagnosis of bare lymphocyte syndrome type I. is made .
;

This patient most likely has mutations in the genes encoding which of the following?

A ) Adenosine deaminase
B ) Fas ligand (CD178)
C ) lnterleukin -2 (IL-2) receptor a chain ( CD 25 )
C_ D ) Peptide transporter (TAP)

o
Previous
o
Next
It?
Lab Values
*
Calculator
o
Review Help Pause
Exam Section 3: Item 43 of 50 National Board of Medical Examiners ^ Time Remaining:
0 Mark Comprehensive Basic Science Self-Assessment 1 hr 12 min S sec

48. A 30 -year -old woman and her 35-year -old husband come to the physician for genetic counseling prior to conception . They have a 2 -year-old daughter with an isolated ventricular septal defect . The wife's sister had a heart defect for which she underwent
operative repair. Previous evaluation of the couple , including echocardiography; showed no abnormalities . Which of the following best represents this couple's risk for having another child with a congenital heart defect?

O A ) 0%
O B ) 3%
O C ) 25%
O D ) 50%
0 E ) 100%

o
Previous
o
Next
tS
Lab Values Calculator
m
Review Help Pause
Exam Section 3: Item 49 of 50 National Board of Medical Examiners ^ Time Remaining:
0 Mark Comprehensive Basic Science Self-Assessment 1 hr 12 min 5 sec

49 . A 70 -year -old woman comes to the physician because of an 3-month history of foul-smelling breath and pain before and after she swallows . Physical examination shows halitosis. Two views from a
barium swallow are shown : the arrow indicates a diverticulum . This abnormality is most likely located between which of the following muscles ?

0 A } Cricopharyngeus and inferior pharyngeal constrictor


6 } Inferior and middle pharyngeal constrictor
C C ) Middle and superior pharyngeal constrictor
D ) Stylopharyngeus and middle pharyngeal constrictor
E } Superior pharyngeal constrictor and stylopharyngeus

o
Previous
o
Next Lab Values Calculator Review
&
Help
tr>
Pause
Exam Section 3: Item SO of 50 National Board of Medical Examiners ^ Time Remaining:
Mark Comprehensive Basic Science Self-Assessment 1 hr 11 min 59 sec

50. A 19-year-old man who is a soldier in the US Army sustains a traumatic brain injury from the detonation of an improvised explosive device while on patrol in the Middle East . This patient is at increased risk for additional neuronal apoptosis caused by diffusion
of which of the following substances from the cerebral capillaries to the brain parenchyma?

O A ) Adenosine
B ) Cortisol
O C ) Glutamate
D ) Lactate
C_ E ) Palmitate

o
Previous
o
Next Lab Values Calculator
m
Review Help Pause
Exam Section 4: Item 1 of 50 National Board of Medical Examiners^ Time Remaining:
Mark Comprehensive Basic Science Self-Assessment 1 hr 14 min 57 sec

1 . Tissue obtained on renal biopsy from a 45- year-old woman with proteinuria is shown . These findings are most consistent with which of the following?

0 A ) History of analgesic abuse


6 } Hypocomplementemia
O C ) Long -standing hyperglycemia
D ) Malignant hypertension
CJ E ) Sickle cell trait

e c* it*
Next Lab Values Calculator Review Help Pause
Exam Section 4: Item 2 of 50 National Board of Medical Examiners ^ Time Remaining:
® Mark Comprehensive Basic Science Self-Assessment 1 hr 14 min 54 sec

2 . A 1 -year-old girl has recurrent and chronic suppurative infections caused by staphylococci and some gram-negative rods. She recovers normally from viral infections. Examination of serum shows normal concentrations of complement components and
immunoglobulins. Studies of leukocyte function are most likely to show defects in which of the following mechanisms ?

O A ) Activation of macrophages
B ) Chemotaxis of macrophages
O C ) Cytotoxic activity of natural killer cells
D ) Ingestion of bacteria by neutrophils
E ) Production of superoxide anions by neutrophils

o
Previous
o
Next
E
(
Lab Values
I!
Calculator
A
Review
&
Help
tl*
Pause
Exam Section 4: Item 3 of 50 National Board of Medical Examiners ^ Time Remaining:
Mark Comprehensive Basic Science Self-Assessment 1 hr 14 min 52 sec

3 . A 57-year-old man with alcoholism has a distended abdomen with shifting dullness and a fluid wave. He has caput medusae palmar erythema and spider angiomata . Which of the following is the most likely additional finding?
; ;

0 A ) Digital clubbing
B ) Enlarged inguinal lymph nodes
0 C ) Flame-shaped retinal hemorrhages
D ) Gynecomastia
0 E ) Jugular venous distention

o
Previous
o
Next
E
(
Lab Values
I!
Calculator
A
Review Help Pause
Exam Section 4: Item 4 of 50 National Board of Medical Examiners ^ Time Remaining:
Mark Comprehensive Basic Science Self-Assessment 1 hr 13 min 36 sec

4 . A 50 -year-old man has had progressive dyspnea on exertion over the past 6 months . He now has dyspnea at rest . He has had a persistent dry cough since having an upper respiratory tract bacterial infection 1 year ago. Examination of lung tissue obtained on
biopsy shows chronic inflammation and fibrous thickening of the alveolar septa . Which of the following is the most likely diagnosis ?

O A ) Chronic bronchitis
B ) Diffuse alveolar damage
O C ) Emphysema
D ) Sarcoidosis
E ) Usual interstitial pneumonitis

o
Previous
o
Next
(E
Lab Values
I!
Calculator
A
Review Help Pause
Exam Section 4: Item 5 of 50 National Board of Medical Examiners ^ Time Remaining:
Mark Comprehensive Basic Science Self-Assessment 1 hr 13 min 33 sec

5 . A pale G 2 -year - old man has had increasing fatigue and indigestion over the past few months and decreasing appetite over the past few weeks . He has moderate splenomegaly but no lyimphadenopathy. Laboratory studies show :
Hemoglobin 9 g/dL
Hematocrit 27%
Leukocyte count 3100/mm 3
Neutrophils 20%
Lymphocytes 75%
Monocytes 5%
Platelet count 75.000/mm 3

The lymphocytes have cytoplasmic projections and positivity for acid phosphatase even in the presence of tartrate . Which of the following is the most likely diagnosis ?

£ ) A ) Acute lymphoblastic leukemia


B ) Acute myelogenous leukemia
O C ) Chronic lymphocytic leukemia
D ) Chronic myelogenous leukemia
E ) Hairy cell leukemia
F ) Infectious mononucleosis

Previous Next
It?
Lab Values
*
Calculator
o
Review Help Pause
Exam Section 4: Item 6 of 50 National Board of Medical Examiners^ Time Remaining:
Mark Comprehensive Basic Science Self-Assessment 1 hr 13 min 28 sec

i
1
O2
Marker 1, 2 1,1
alleles

II P
1 2 3
Marker 1,1 1,1 1, 2
alleles

0 Affected female
Affected male
Pregnancy, sex unspecified
O Unaffected female
Unaffected male

6 . A 25 -year-old woman who is at 12 weeks ' gestation asks for prenatal counseling . Her husband has a clinically variable autosomal dominant disorder with 100% penetrance that is evident within the first year of life and for which linkage analysis is available.
They have two unaffected daughters 3 and 5 years old . A pedigree is shown . A single polymorphic marker has a recombination fraction of 0.1 ( 10%) and is informative . Alleles 1 and 2 for this marker are indicated in the pedigree . Which of the following is the
risk that this fetus will have this disorder ?

O A ) 1%
O B ) 10%
O c ) 50%
O D ) 90%
E ) 99%

o
Previous
o
Next
e
Lab Values Calculator
9
Review
0
Help
0
Pause
Exam Section 4: Item 7 of 50 National Board of Medical Examiners ^ Time Remaining:
Mark Comprehensive Basic Science Self-Assessment 1 hr 13 min 25 sec

7 . A 24-year-old woman comes to the clinic because of facial flushing and diffuse pruritus .. The symptoms began shortly after she ingested a new nutritional supplement from a health food store . Examination shows erythema of the face and upper portion of the
neck . The remainder of the skin shows no gross abnormalities . There is no perioral or neck edema . Findings on lung examination are normal. Which of the following nutritional supplements is the most likely cause of her symptoms ?

A ) Ascorbic acid
B ) Folic acid
C ) Lipoicacid
D ) Nicotinic acid
E ) Pantothenic acid

o
Previous
o
Next
E
(
Lab Values
I!
Calculator
A
Review Help Pause
Exam Section 4: Item 8 of 50 National Board of Medical Examiners^ Time Remaining:
Mark Comprehensive Basic Science Self-Assessment 1 hr 13 min 23 sec

8 . A 2 -week-old boy is brought to the physician because of persistent cyanosis . Hemoglobin concentration and erythrocyte morphology are normal An x-ray of the chest and an echocardiogram disclose no abnormalities . Which of the following is the most likely
cause of the cyanosis ?

A ) Hemolytic anemia
B ) Megaloblastic anemia
O C ) Methemoglobinemia
D ) a- Thalassemia
E ) p -Thalassemiia

o
Previous
o
Next Lab Values Calculator
m
Review
c*
Help
it*
Pause
Exam Section 4: Item 9 of 50 National Board of Medical Examiners ^ Time Remaining:
Mark Comprehensive Basic Science Self-Assessment 1 hr 13 min 20 sec

9 . A 63-year-old woman comes to the physician because of fatigue and weakness for the past 3 months . She has not had abdominal pain or a change in bowel habits . Findings on physical examination are unremarkable. Laboratory studies show :
Hemoglobin 9.5 g / dL
Hematocrit 29%
Mean corpuscular volume 70 pm 3
Mean corpuscular hemoglobin 20 pg/cell

Adenocarcinoma of the colon is suspected . Which of the following best describes the most likely location of the primary tumor in the large intestine ?

A ) Ascending
B ) Descending
O C ) Rectum
D ) Splenic flexure
E ) Synchronous multiple sites
F ) Transverse

o
Previous
o
Next Lab Values Calculator
m
Review
C*
Help
it*
Pause
Exam Section 4: Item 10 of 50 National Board of Medical Examiners^ Time Remaining:
Mark Comprehensive Basic Science Self-Assessment 1 hr 13 min IT sec

10. A 62 -year-old woman has an abdominal aortic aneurysm repaired . Forty-eight hours after surgery her right distal leg is dusky and cool and there are dark purple-to -black necrotic lesions on several toes as shown . A biopsy
; ,

of one of the lesions would most likely show occluded small arteries with needle-shaped clefts. Which of the following is the most likely cause of the lesions ?

0 A ) Arteriolar narrowing
6 } Cholesterol emboli
O C ) Septic emboli
D ) Vasospasm
JJ

E } Venous thrombosis

o
Previous
o
Next Lab Values
!
I
Calculator
A
Review
m
Help
o
Pause
Exam Section 4: Item 11 of 50 National Board of Medical Examiners^ Time Remaining:
Mark Comprehensive Basic Science Self-Assessment 1 hr 13 min 14 sec

11 . A 72 -year - old man comes to the physician because of a 1 -month history of discoloration of his legs. He lives alone and his diet consists mostly of oatmeal and prepared meats. Physical examination shows ecchymoses of the lower extremities , splinter
hemorrhages of the nailbeds, perifollicular purpura and corkscrew hairs. A deficiency of which of the following vitamins is most likely responsible for these findings?
,

.... A) A
'
,
B ) B (thiamine)
o C ) Be (pyndoxine)
;

O D) c
O E) E

o
Previous
o
Next Lab Values
I!
Calculator
A
Review
m
Help
o
Pause
Exam Section 4: Item 12 of 50 National Board of Medical Examiners^ Time Remaining:
Mark Comprehensive Basic Science Self-Assessment 1 hr 13 min 12 sec

12 . A 29-year-old woman has had three spontaneous abortions between the third and fourth weeks of gestation . The most likely cause is failure of her ovaries to produce sufficient quantities of which of the following hormones ?

0 A ) Estrogen
B ) Human chorionic gonadotropin
0 C ) Luteinizing hormone
D ) Oocyte maturation inhibitor
0 E ) Progesterone

o
Previous
o
Next
E
(
Lab Values
!
I
Calculator
A
Review
m
Help
o
Pause
Exam Section 4: Item 13 of 50 National Board of Medical Examiners^ Time Remaining:
0 Mark Comprehensive Basic Science Self-Assessment 1 hr 13 min 7 sec

13. A 3G -year-old woman is being evaluated for infertility. Menarche occurred at age 12 years and she had regular menses until approximately 3 years ago . At that time, she noticed increased spacing between her menstrual periods and decreasing flow . Physical
;

examination shows no abnormalities . Laboratory findings show normal serum testosterone and prolactin concentrations and an increased serum follicle -stimulating hormone concentration . Which of the following is the most likely cause of the amenorrhea ?

O A ) Endometriosis
B ) Hypothyroidism
C ) Pituitary tumor
D ) Polycystic ovarian syndrome
E ) Premature ovarian failure

o
Previous
o
Next Lab Values
7=
Calculator
m
Review Help Pause
Exam Section 4: Item 14 of 50 National Board of Medical Examiners^ Time Remaining:
Comprehensive Basic Science Self-Assessment 1 hr 13 min 5 sec

injection injection injection injection injection injection


7i l I
l I
J I

. .J
I ‘••I
•,*&
* 'M
<D
-
/ J*.
/ t .
o

o 0
O
/
* O
'
LO
£
=
Ci3
CL o
* •O
0 Vl
!IJo
* *'

D =^
/>
*
*
r'Si\
I O
5- O
I
4

I
I
I
I
I
4H f
0 2 4 6 0 10
--
<> <3 o Active drug + Saline Treatment week
*—*-* Saline

14 . A study is conducted to assess the effectiveness of injections of lidocaine into "trigger points" of pain symptoms in patients with fibromyalgia . Fifty patients are randomly assigned to biweekly trigger - point injections with 0.9% saline only or 0.9 % saline plus
lidocaine. The graph shows daily self-reported pain scores in relation to the injections . Which of the following is the most likely explanation for these findings ?

O A ) Placebo effect
B ) Regression to the mean
O C ) Selection bias
D ) Type II error
! E ) Uncontrolled confounding

o
Previous
o
Next
If!
Lab Values
e
Calculator Review Help Pause
Exam Section 4: Item 15 of 50 National Board of Medical Examiners^ Time Remaining:
0 Mark Comprehensive Basic Science Self-Assessment 1 hr 13 min 2 sec

15. A 34-year-old woman comes to the burn unit where her 14 -year - old son was admitted 2 weeks ago after he injured his hand while lighting firecrackers with his friends . In preparation for his discharge the woman is taught to change the dressing and apply
;

cream to the burn. She says that she feels revulsion when viewing the affected area and changing the dressing but she tries to look cheerful for the sake of her son . Which of the following best describes this mother's defense mechanism?
;

CJ A ) Denial
B ) Displacement
C ) Dissociation
D ) Isolation of affect
E ) Repression
F ) Splitting
CJ G ) Suppression

o
Previous
o
Next
tS
Lab Values Calculator
m
Review Help Pause
Exam Section 4: Item 16 of 50 National Board of Medical Examiners^ Time Remaining:
0 Mark Comprehensive Basic Science Self-Assessment 1 hr 13 min 0 sec

16. A 17 -year-old boy comes to the physician for a follow-up examination . One year ago he sustained a deep laceration to his right lower extremity in a motorcycle collision . He underwent operative repair of a severed neurovascular bundle below the popliteal
;

fossa. During the next 10 months, he received regular physical therapy. Physical examination now shows no apparent atrophy of the right lower extremity. Motor and sensory functions are intact . Which of the following changes is most likely in the muscle
fibers of the patient's right foot?

A ) Chronic inflammation
B ) Degeneration
O C ) Fibrosis
D ) Grouping by fiber type
E ) Pseudohypertrophy

o
Previous
o
Next Lab Values Calculator
m
Review Help Pause
Exam Section 4: Item 17 of 50 National Board of Medical Examiners^ Time Remaining:
Mark Comprehensive Basic Science Self-Assessment 1 hr 12 min 57 sec

17. The initial event in muscle contraction is the release of calcium ion from the sarcoplasmic reticulum. Which of the following is most likely responsible for buffering the concentration of calcium ion in the sarcoplasmic reticulum?

0 A ) Calcium -dependent protein kinase


B ) Calmodulin
0 C ) Calsequestrin
D ) Myosin light chain
0 E ) Troponin

o
Previous
o
Next
(E
Lab Values
!
I
Calculator
A
Review
m
Help
o
Pause
Exam Section 4: Item 13 of 50 National Board of Medical Examiners^ Time Remaining:
Mark Comprehensive Basic Science Self-Assessment 1 hr 12 min 53 sec

18. A study is conducted to assess the relationship between a history of cigarette smoking and the development of transitional cell carcinoma. In 1G 00Q patients with transitional cell carcinoma of the bladder 6750 had a history of cigarette smoking : in 1 Q 000
: ;

patients without transitional cell carcinoma . 1250 had a history of cigarette smoking . Which of the following best describes this study design?

A ) Case-control study
B ) Case series study
C ) Cohort study
D ) Cross-sectional study
J
^ E } Randomized control study

o
Previous
o
Next
(E
Lab Values
I!
Calculator
A
Review Help Pause
Exam Section 4: Item 19 of 50 National Board of Medical Examiners ^ Time Remaining:
Comprehensive Basic Science Self-Assessment 1 hr 12 min 49 sec

19. A 4-year-old boy undergoes radiographic imaging studies of the urinary tract after having two bacterial urinary infections during the past year . His left kidney is found to be abnormally small and
nonfunctional . The right kidney appears normal . He undergoes a left nephrectomy and ureterectomy. A photograph of the resected specimen is shown . Which of the following disorders most likely
accounts for the pathologic changes in this kidney?

C A ) Arteriolonephrosclerosis
6 } Hypospadias
C C ) Panhypopituitarism
D ) Renal papillary necrosis
G E } Ureteral obstruction

o
Previous
o
Next Lab Values Calculator
<r
Review Help Pause
Exam Section 4: Item 20 of 50 National Board of Medical Examiners^ Time Remaining:
® Mark Comprehensive Basic Science Self-Assessment 1 hr 12 min 45 sec

20. A 65 -year -old man is brought to the emergency department 12 hours after the onset of chest pain . He dies 5 days later of cardiogenic shock . At autopsy examination of the heart shows blockage of the coronary arteries and massive amounts of ventricular
,

necrosis. A lack of which of the following best explains the inability of this patient’s heart muscle to repair itself?

A ) Myoglobin
B ) Sarcolemma
O C ) Sarcoplasmic reticulum
D ) Satellite cells
E ) T tubules

o
Previous
o
Next
(E
Lab Values
I!
Calculator
A
Review Help Pause
Exam Section 4: Item 21 of 50 National Board of Medical Examiners^ Time Remaining:
Comprehensive Basic Science Self-Assessment 1 hr 12 min 41 sec

21. A 50 -year -old man is admitted to the hospital after being diagnosed with a pulmonary embolus . Treatment is started with intravenous heparin. Twenty-four hours later warfarin is added . On day 2 his partial thromboplastin time is 52 seconds ( control 26 sec ) ,
;

and prothrombin time is 12 seconds (control 12.1 sec : INR=1) Which of the following is the best explanation for the normal prothrombin time and INR measurements in this patient?

O A ) Heparin-warfarin interaction
B ) Hereditary resistance to heparin
O C ) Hereditary resistance to warfarin
D ) Long half-life of factor II (prothrombin)
E ) Too low a dose of heparin
F ) Too low a dose of warfarin
G ) Undetected liver disease

o
Previous
o
Next Lab Values Calculator
m
Review Help Pause
Exam Section 4: Item 22 of 50 National Board of Medical Examiners^ Time Remaining:
Mark Comprehensive Basic Science Self-Assessment 1 hr 12 min 38 sec

22 . A 35 -year -old man conies to the physician because of progressively painful erections during the past year. Examination of the penis shows abnormal curvature shortening on erection and a subcutaneous plaque on the dorsal surface . Which of the following
, ,

is the most likely diagnosis ?

O A ) Desmoid tumor
B ) Dupuytren contracture
C ) Keloids
D ) Nodular fasciitis
E ) Peyronie disease

o
Previous
o
Next
E
(
Lab Values
I!
Calculator
A
Review Help Pause
Exam Section 4: Item 23 of 50 National Board of Medical Examiners^ Time Remaining:
Mark Comprehensive Basic Science Self-Assessment 1 hr 12 min 33 sec

23. A 27 -year -old man is admitted to the hospital because of fever fatigue: malaise anemia , and lymphadenopathy. He is a marine and had been stationed in Saudi Arabia . He has a history of a lesion that developed at the site of a sand fly bite 2 weeks before
;

his current symptoms began . Blood cultures are negative . A biopsy specimen of a skin lesion shows enlarged macrophages containing numerous intracellular organisms . Which of the following is the most likely causal organism?

O A ) Dracunculus medinensis
B ) Leishmanta tropica
C ) Onchocerca volvulus
D ) Plasmodium ovale
E ) Trypanosoma brucei

o
Previous
o
Next Lab Values
*
Calculator
m
Review Help Pause
Exam Section 4: Item 24 of 50 National Board of Medical Examiners^ Time Remaining:
Mark Comprehensive Basic Science Self-Assessment 1 hr 12 min 33 sec

24 . A study is conducted to examine the effects of polychemotherapy on patients with lung carcinoma . Bronchoalveolar lavage samples taken before and after treatment with methotrexate , doxorubicin cyclophosphamide , and lomustine are examined for changes
,

in pulmonary surfactant . In order to access a lingular bronchus for lavage, the bronchoscope must passthrough which of the following bronchi ?

C _) A ) Left lower lobe


B ) Left upper lobe
O C ) Right lower lobe
D ) Right middle lobe
E ) Right upper lobe

o
Previous
o
Next Lab Values
4=
Calculator
m
Review
c*
Help
it*
Pause
Exam Section 4: Item 25 of 50 National Board of Medical Examiners^ Time Remaining:
Mark Comprehensive Basic Science Self-Assessment 1 hr 12 min 30 sec

25. A 7-year-old boy is brought to the physician because of a 3- day history of cramping abdominal pain blood in his urine, and a rash on his arms, legs, and buttocks . He had pharyngitis 2 weeks ago. Physical examination shows a hemorrhagic rash involving the
:

extensor surfaces of the upper and lower extremities and feet . There is generalized abdominal tenderness . Test of the stool for occult blood is positive . A renal biopsy specimen shows an increased mesangial matrix that stains positive for IgA on
immunofluorescence. Which of the following is the most likely diagnosis?

A ) Diabetic glomerulosclerosis
B ) Focal segmental glomerulosclerosis
C ) Henoch- Schonlein purpura
D ) Minimal change disease
E ) Nephroblastoma (Wilms tumor)
F ) Poststreptococcal glomerulonephritis
G ) Renal amyloidosis

o
Previous
o
Next
It?
Lab Values
*
Calculator
®
Review
r
Help
r
Pause
Exam Section 4: Item 26 of 50 National Board of Medical Examiners^ Time Remaining:
Mark Comprehensive Basic Science Self-Assessment 1 hr 12 min 28 sec

26. A 33-year -old nulligravid woman comes to the physician with her 35-year -old husband because she has not been able to conceive for 13 months. Previous evaluation of the husband showed no abnormalities Physical and pelvic examinations show no
abnormalities . Examination of an endometrial biopsy specimen shows secretory glands and a decidualized stroma . Which of the following hormonal changes underlies the maturation of this patient' s endometrium?

O A ) Decreased follicle-stimulating hormone from the pituitary gland


B ) Decreased luteinizing hormone from the pituitary gland
O C ) Decreased production of testosterone from the adrenal cortical cells
D ) Increased estradiol from the follicle
E ) Increased p-human chorionic gonadotropin from the ovarian theca cells
F ) Increased progesterone from the corpus luteum

o
Previous
o
Next
£5
Lab Values
e
Calculator
©
Review
&
Help Pause
Exam Section 4: Item 27 of 50 National Board of Medical Examiners^ Time Remaining:
Mark Comprehensive Basic Science Self-Assessment 1 hr 12 min 23 sec

27. A 1 -year-old girl is brought to the physician because of multiple bacterial infections since birth . She is at the 40 th percentile for length and 40th percentile for weight . She has white hair pale skin blue iriises and prominent red pupils. A compete blood count
; ;

shows neutropenia. A peripheral smear shows giant granules in the neutrophils. This patient’s disorder is most likely due to which of the following pathogenetic mechanisms?

O A ) Bone marrow suppression


B ) Defect in leukocyte adhesion
O C ) Defect in phagolysosome function
D ) Myeloperoxidase deficiency
' _ ! E } NADPH oxidase deficiency

o
Previous
o
Next
E
(
Lab Values
!
I
Calculator
A
Review
&
Help
tl*
Pause
Exam Section 4: Item 23 of 50 National Board of Medical Examiners^ Time Remaining:
Mark Comprehensive Basic Science Self-Assessment 1 hr 12 min 21 sec

28. A 25 -year -old woman comes to the physician because of a 3 -month history of difficulty falling and remaining asleep. She says that she has been under stress and anxious during this period . Physical examination shows no abnormalities . Nocturnal
polysomnography shows normal sleep architecture including normal volumes and latencies of all non-REM and REM sleep stages. There is no evidence of sleep -disordered breathing or other sleep -fragmenting disorder. Compared with this patient's non -
;

REM stage 3 to 4 (deep ) sleep which of the following is most likely to be increased in her REM sleep ?
;

A ) Cerebral blood flow


B ) Delta wave EEG activity
C ) Intercostal muscle activity
D ) Serum growth hormone concentrations
E ) Thermoregulation

o
Previous
o
Next
(£5
Lab Values
e
Calculator
©
Review
&
Help Pause
Exam Section 4: Item 29 of 50 National Board of Medical Examiners ^ Time Remaining:
Mark Comprehensive Basic Science Self-Assessment 1 hr 12 min IT sec

29. During ligand-binding experiments , a new adrenergic drug displays potent binding to a-adrenergic receptors. In healthy volunteers: the drug causes a significant increase in blood pressure . However when used in the emergency department to increase blood
pressure in trauma victims, the drug decreases blood pressure . This drug is most likely from which of the following classes?

O A ) Competitive antagonist
B ) Full agonist
O C ) Inverse agonist
D ) Noncompetitive antagonist
E ) Partial agonist

o
Previous
o
Next Lab Values Calculator
m
Review
c*
Help
it*
Pause
Exam Section 4: Item 30 of 50 National Board of Medical Examiners ^ Time Remaining:
Mark Comprehensive Basic Science Self-Assessment 1 hr 12 min IS sec

30. A 70 -year-old man dies of coronary artery disease . He had a cerebral infarction 8 years ago. A photograph of a specimen of the brain stem is shown . Which of the following neurologic deficits was most likely I gff
present in this patient after his cerebral infarction?

0 A } Dysmetria on the left


B } Dysmetria on the right
C C ) Loss of joint position sense in the left extremities
D ) Loss of joint position sense in the right extremities
E ) Spastic hemiparesis on the left
F } Spastic hemiparesis on the right

o
Previous
o
Next Lab Values Calculator
m
Review
r
Help *r*
Pause
Exam Section 4: Item 31 of 50 National Board of Medical Examiners ^ Time Remaining:
0 Mark Comprehensive Basic Science Self-Assessment 1 hr 12 min 5 sec

31. A 14-year -old girl is brought to the physician because of a recent growth spurt of 15 cm (6 in ) during the past year. She also has had increasing fatigue and palpitations during this period . Her father is 193 cm (6 ft 4 in) Her paternal aunt has a history of
palpitations and severe myopia. The patient is at the 95th percentile for height and 50th percentile for weight Physical examination shows a long thin face . Ophthalmologic examination shows dislocated lenses . Cardiac examination shows a hyperdynamic
,

precordium with early click and systolic murmur . Echocardiography shows an enlarged aortic root and mitral valve prolapse . Which of the following genetic mechanisms best explains the findings in this family?

A ) Anticipation
B ) Heterogeneity
C ) Paternal disomy
D ) Pleiotropy
E ) Polygenes

o
Previous
o
Next
It?
Lab Values
*
Calculator
®
Review
f
Help Pause
Exam Section 4: Item 32 of 50 National Board of Medical Examiners^ Time Remaining:
0 Mark Comprehensive Basic Science Self-Assessment 1 hr 12 min 3 sec

32 . A 46 -year -old man receives the diagnosis of squamous cell carcinoma of the esophagus. A barium swallow is shown . Esophagectomy at the region indicated by the arrows is most likely to involve ligation of
arterial branches of which of the following vessels ?

0 A ) Aorta
6 } Internal thoracic arteries
C C ) Pulmonary arteries
D ) Superior mesenteric artery
JJ

E } Thyrocervical trunk

o
Previous
r
Next Lab Values Calculator
m
Review
C*
Help
tr>
Pause
Exam Section 4: Item 33 of 50 National Board of Medical Examiners ^ Time Remaining:
Comprehensive Basic Science Self-Assessment 1 hr 12 min 0 sec

33. A 50-year-old man with chronic back pain has taken a variety of analgesics (including aspirin acetaminophen , and ibuprofen) daily and in large doses for the past 25 years. He is found dead . A photograph of his kidneys at autopsy is shown . Prior to his death ,
,

which of the following changes in urine volume and urine osmolality would be expected after administration ofADH (vasopressin ) ?
Urine Volume Urine Osmolality
G A> Increased increased
G B> Increased no change
Oc> Increased decreased
G D) No change increased
OE) No change no change
On No change decreased
O G) Decreased increased
OH) Decreased no change
O)
' Decreased decreased

o
Previous
o
Next Lab Values Calculator
m
Review Help Pause
Exam Section 4: Item 34 of 50 National Board of Medical Examiners^ Time Remaining:
Mark Comprehensive Basic Science Self-Assessment 1 hr 11 min 57 sec

34 . A 72 -year -old man with multiple myeloma agrees to participate in an investigational chemotherapy clinical trial. Prior to enrollment in the study peripheral blood B and T lymphocytes are isolated and undergo Southern blot analysis of lymphoid cell-specific
;

genes. Using a single B - lymphocyte J-region probe analysis of the B -lymphocyte DNA shows a 1.5- kb band. Analysis of the T-lymphocyte DNA using the same J-region probe shows a 6-kb band . The 6-kb band most likely signifies which of the following?
:

O A ) Constant region gene rearrangement


B ) D to J gene rearrangement
O C ) V to 0 gene rearrangement
D ) V to J gene rearrangement
' _ ! E ) Unrearranged immunoglobulin gene

o
Previous
o
Next Lab Values Calculator
m
Review
f*
Help
it*
Pause
Exam Section 4: Item 35 of 50 National Board of Medical Examiners ^ Time Remaining:
® Mark Comprehensive Basic Science Self-Assessment 1 hr 11 min 54 sec

35. AG- month-old girl is brought to the emergency department by her parents because of vomiting for 6 hours. She has been feeding poorly for 1 day because of an upper respiratory tract infection . Her temperature is 37° C ( 98.6° F) . pulse is 100/min respirations
:

are 13/min . and blood pressure is SS/5 S mm Hg . Physical examination shows lethargy and a clear nasal discharge . Laboratory studies show :
Serum
Ammonia increased
Citrulline decreased
Glutamine increased
Urine orotic acid increased

This patient most likely has a deficiency of which of the following enzyme activities ?

O A ) Arginase
B ) Argininosuccinase
O C ) Arg ini nosuccinate synthetase
D ) Carbamoyl phosphate synthetase
E ) Ornithinetranscarbamylase

o
Previous
o
Next Lab Values Calculator
m
Review Help Pause
Exam Section 4: Item 36 of 50 National Board of Medical Examiners^ Time Remaining:
Mark Comprehensive Basic Science Self-Assessment 1 hr 11 min 51 sec

36. A 53 -year -old woman comes to the physician because of a 3 -week history of shortness of breath after walking 2 blocks. She has been receiving doxorubicin as part of chemotherapy for breast cancer for 6 months . Her pulse is 110/min: and respirations are
32/min Inspiratory crackles are heard over both lung fields. Physical examination shows 2+ pitting edema of the ankles. A chest x -ray shows cardiomegaly and interstitial edema . Cardiac studies show a decreased ejection fraction of the left ventricle. Which of
the following is the most likely cause of the cardiac findings in this patient?

O A ) Decreased oxidative phosphorylation


B ) Impairment of [3-oxidation of fatty acids
C ) Inhibition of acetylcholinesterase activity
D ) Inhibition of metal-dependent enzyme synthesis
! _ E ) Lipid peroxidation of myocyte membranes

Previous Next
It?
Lab Values
*
Calculator
o
Review Help Pause
Exam Section 4: Item 37 of 50 National Board of Medical Examiners ^ Time Remaining:
® Mark Comprehensive Basic Science Self-Assessment 1 hr 11 min 49 sec

37. A 14-year -old boy is brought to the emergency department by his friend because of palpitations. His friend says the symptoms began shortly after he "snorted" cocaine. He is found to have tachycardia and increased mean arterial pressure. Which of the
following mechanisms is the most likely cause of these adverse effects ?

3 A ) Agonist action at ligand-gated Ga 2^ channels


3 B } Agonist action at ligand-gated Na -K - channels
+
' 4

O C ) Inhibition of cardiac M ^receptor channels


D ) Inhibition of norepinephrine reuptake
3 E ) Inhibition of phosphodiesterase

o
Previous
o
Next Lab Values Calculator
m
Review Help Pause
Exam Section 4: Item 33 of 50 National Board of Medical Examiners^ Time Remaining:
Mark Comprehensive Basic Science Self-Assessment 1 hr 11 min 47 sec

38. A 32 -year -old woman undergoes a laparoscopic cholecystectomy after ultrasonography showed a thickened gallbladder wall and gallstones. The surgeon will be unable to inspect directly which of the following organs during this procedure ?

A ) Ileum
B ) Jejunum
O C ) Pancreas
D ) Stomach
! E ) Transverse colon

o
Previous
o
Next Lab Values Calculator
m
Review
c*
Help
it*
Pause
Exam Section 4: Item 39 of 50 National Board of Medical Examiners ^ Time Remaining:
® Mark Comprehensive Basic Science Self-Assessment 1 hr 11 min 44 sec

39. A 4-year-old girl is brought to the physician by her parents 2 days after the mother noticed streaks of blood on the child' s well -formed stools . Her father has pigmented macules on his lips. A paternal aunt had breast cancer. Physical examination shows several
2 - to 3-mm pigmented macules on the lips and buccal mucosa. Rectal examination shows no abnormalities . Test of the stool for occult blood is positive. Colonoscopy is done and three polyps are removed the largest measuring 1 x 1 x 1 cm . This patient's
,

polyps are most likely which of the following types?

A ) Adenomatous
B ) Hamartomatous
C ) Hyperplastic
D ) Inflammatory
E ) Lymphoid

o
Previous
o
Next Lab Values Calculator
m
Review Help Pause
Exam Section 4: Item 40 of 50 National Board of Medical Examiners^ Time Remaining:
Mark Comprehensive Basic Science Self-Assessment 1 hr 11 min 41 sec

40. An SG-year - old woman is brought to the emergency department by police 30 minutes after she was found wandering in the street, confused and disoriented . She says that she has no medical conditions and takes no medications. During history taking, she
becomes annoyed and agitated saying: " Stop being so nosey ! There's nothing wrong with me ! I feel fine and I just want to go home ." She appears frail but alert . Her vital signs are within normal limits. Physical examination shows mildly dry mucous
;

membranes There is no evidence of trauma . She is oriented to person but not to place or time. She is able to provide her home address . Which of the following is the most appropriate action by the physician?

A ) Attempt to contact the patients family to find out more information


B } Arrange for transportation to take the patient home
C ) Arrange for the patient to be transferred to a skilled nursing care facility
D ) Administer a sedative- hypnotic medication
! _ E ) Admit the patient to a psychiatric inpatient facility

o
Previous
o
Next
It?
Lab Values
*
Calculator
o
Review Help Pause
Exam Section 4: Item 41 of 50 National Board of Medical Examiners^ Time Remaining:
Comprehensive Basic Science Self-Assessment 1 hr 11 min 39 sec

41 . A 73-year -old man conies to the physician because of a 4-month history of localized mid- back pain . He has hypertension and type 2 diabetes mellitus. He is in mild discomfort but is able to walk without problems . There is pain with percussion over the lower
thoracic spine. Neurologic examination shows no focal findings. Serum studies show:
Ca 2+ 9.2 mg/dL
Phosphorus 3.9 mg/dL
Total bilirubin 0.5 mg/dL
Alkaline phosphatase 1145 U/L
AST 34 U/L
ALT 23 U/L
y-GlutamyItransferase ( GGT ) 26 U/L (N=0—30 }

X -rays of the spine show vertebral cortical thickening of the end - plates creating ''picture frame" vertebrae. Technetium 99m scan shows increased areas of uptake in the skull , thoracic spine lumbar spine , and right proximal femur. Which of the following best
; ;

describes the primary underlying pathophysiologic mechanism of this condition?

1I A ) Decreased number and activity of osteoblasts


B ) Decreased production of interleukin-6 ( IL-6)
C) C ) Decreased synthesis and secretion of intact parathyroid hormone
D ) Increased number and activity of osteoclasts
E ) Increased production of IL- 12

o
Previous
o
Next
If!
Lab Values
e
Calculator
©
Review
&
Help Pause
Exam Section 4: Item 42 of 50 National Board of Medical Examiners^ Time Remaining:
Mark Comprehensive Basic Science Self-Assessment 1 hr 11 min 37 sec

42 . A 5-year-old boy is brought to the physician by his mother because of progressive clumsiness and fatigue during the past 6 months . He says that his legs are tired . He was delivered at term after an uncomplicated pregnancy. He has met all developmental
milestones, although there was some delay compared with other children his age . He is alert . He has difficulty rising from the chair : he uses his arms to push himself into a standing position . He is unable to jump with both feet together. Physical examination
shows hypertrophy of the calf muscles. This patient most likely has weak hip adduction as a result of dysfunction of the muscle inserting onto the femur from which of the following locations ?

A ) Anterior sacrum
B ) Iliac crest
C ) Iliac spine
D ) Ischium
E ) Lateral ilium

o
Previous
o
Next
It?
Lab Values
*
Calculator
®
Review
r
Help
r
Pause
Exam Section 4: Item 43 of 50 National Board of Medical Examiners^ Time Remaining:
® Mark Comprehensive Basic Science Self-Assessment 1 hr 11 min 34 sec

43. During an experiment , Drug X is administered to treat trematode and cestode infestations. Results show that Drug X increases the permeability of the cell membranes to calcium, causing paralysis, dislodgement, and death of the parasite . Drug X most closely
resembles which of the following?

A ) Albendazole
B ) Bithionol
O C ) Diethylcarbamazine
D ) Niclosamide
E ) Praziquantel

o
Previous
o
Next Lab Values Calculator
m
Review Help Pause
Exam Section 4: Item 44 of 50 National Board of Medical Examiners^ Time Remaining:
Mark Comprehensive Basic Science Self-Assessment 1 hr 11 min 31 sec

44 . Investigators are evaluating oxygen consumption by different segments of the renal tubules in experimental animals. Arterial oxygen saturation is decreased in blood delivered to isolated perfused pig kidneys. Results show that as oxygen delivery declines:
urine output decreases . A biopsy specimen of the ischemic model is obtained . Tubular cell death is most likely to be observed in which of the following areas ?

O A ) Cortical collecting duct


B ) Distal convoluted tubule
C ) Loop of Henle
D ) Papilla
E ) Proximal convoluted tubule

o
Previous
o
Next
tS
Lab Values Calculator
m
Review Help Pause
Exam Section 4: Item 45 of 50 National Board of Medical Examiners^ Time Remaining:
Mark Comprehensive Basic Science Self-Assessment 1 hr 11 min 29 sec

45. A 33-year -old man conies to the physician for a physical examination prior to starting a new job . He feels well and recently became engaged to be married. Physical examination shows a 1 5-cm fixed nontender mass on the right side of the neck . During the
,
;

examinatiom he appears anxious and says: MYou seem to be spending a lot of time examining my neck . Is there something wrong? Do I have cancer ?' After informing the patient about the discovery of the suspected mass, which of the following is the most
1

appropriate initial statement by the physician?

A ) "I know that you’re anxious about this, so I think iit would be a good idea for your fiancee to come in with you when you return for a biopsy next week . "
B ) " It ' s understandable that you're worried , so I' ll walk you through each step. We' ll need to schedule a time as soon as possible to get a sample of the mass for further examination ."
C ) " The only way to know if the mass in your neck is malignant or benign is to do a biopsy. Please take this referral to the front desk so that they can schedule the procedure and a follow- up visit with me."
D ) "We’ll need to do a biopsy of the mass to determine if it is malignant or benign . Until we have the results of the biopsy I don ' t think you should worry about it ."
E ) "Whatever this mass turns out to be it'll be easier to face because you’ll have the support of your fiancee . "
:

o
Previous
o
Next
If!
Lab Values
e
Calculator
©
Review
&
Help Pause
Exam Section 4: Item 46 of 50 National Board of Medical Examiners^ Time Remaining:
Mark Comprehensive Basic Science Self-Assessment 1 hr 11 min 26 sec

46. An investigator is studying the regulation of pulmonary lymphatic flow using an animal model. Catheters are implanted in the femoral vein pulmonary artery and main lymphatic vessel draining the lungs in anesthetized . intubated mechanically ventilated
,
; ;

animals. Drugs may be injected directly into the pulmonary artery catheter and inspired gas mixtures altered at the ventilator ; timed collection of lymph from the catheters is used to calculate lymphatic flow. Which of the following interventions will most likely
increase the flow of pulmonary lymph in these animals?

A ) Administration of erdothelin-1 into the pulmonary artery


B ) Administration of phenylephrine into the pulmonary artery
C ) Decreasing the inspired oxygen concentration from 21% to 10%
D ) Increasing the inspired carbon dioxide concentration from 0.3% to 3%
E ) Intravenous infusion of 0.9% saline for 5 minutes
F ) Intravenous infusion of 20 % albumin solution (20 g/100 ml_ saline) for 5 minutes

o
Previous
o
Next
£5
Lab Values
e
Calculator
©
Review
&
Help Pause
Exam Section 4: Item 47 of 50 National Board of Medical Examiners ^ Time Remaining:
Mark Comprehensive Basic Science Self-Assessment 1 hr 11 min 24 sec

47. An investigator is studying the thyroid hormone receptor. It is found that the receptor binds chromatin in both the presence and absence of triiodothyronine (T 3). Results show that this binding inhibits transcription in the absence of the hormone and activates
transcription in the presence of the hormone . It is theorized that binding of the hormone causes an allosteric conformational change that decreases affinity for complex 1 and increases affinity for complex 2 . Complex 1 and complex 2 most likely contain which
of the following enzyme activities?

Complex 1 Complex 2

O A) Deiodinase thyroxine peroxidase


O B) Histone deacetylase histone acetyltransferase
O C) Restriction endonuclease DNA ligase
O D) Tyrosine kinase protein tyrosine phosphatase
O E) Ubiquitin ligase SUMO ligase

o
Previous
o
Next
It?
Lab Values
*
Calculator
e
Review
r
Help
r
Pause
Exam Section 4: Item 43 of 50 National Board of Medical Examiners^ Time Remaining:
Comprehensive Basic Science Self-Assessment 1 hr 11 min 21 sec

48. A 75 -year -old woman with chronic obstructive pulmonary disease conies to the physician because of a 3- month history of shortness of breath while performing household chores and swelling of her feet at night . She smoked 2!4 packs of cigarettes daily for
50 years until she decreased to one-half pack daily 2 years ago . She takes no medications and does not use supplemental oxygen . Her respirations are 18/ min and labored . Physical examination shows cyanosis of the lips and fingertips. Crackles rhonchi .
;

and wheezing are heard over all lung fields bilaterally. There is a markedly prolonged expiration phase. Arterial blood gas analysis on room air is most likely to show which of the following sets of findings in this patient?

pH Po2 { mm Hg) Pco 2 ( mm Hg ) HC 03- ( mEq/ L)


O A) 7.32 39 43 24
O B) 7.35 65 56 30
O C) 7.39 94 39 24
O D) 7.41 62 43 32
O E) 7 42 72 32 21

O
Previous
O
Next Lab Values
*
Calculator
o
Review Help Pause
Exam Section 4: Item 49 of 50 National Board of Medical Examiners^ Time Remaining:
Mark Comprehensive Basic Science Self-Assessment 1 hr 11 min IS sec

Coronal Axial
49. A 78 -year -old man who is a Vietnam War veteran comes to the physician because of a 3- month history of low back pain that has become increasingly severe during the past 2 weeks and has caused difficulty walking because of pain radiating down his left
,

leg . He has a 3-week history of night sweats and an intermittent low- grade fever. He also has had a 9-kg (20 - lb) weight loss during the past 3 months. He appears ill . He is 173 cm (5 ft 8 in) tall and weighs 55 kg (121 lb ) ; BMI is 18 kg /m 2 His temperature is
38.1°C ( 10 D . GaF) and pulse is 85/ min . Physical examination shows pallor right convex lumbar scoliosis , and tenderness over the L2 -3 area . He has difficulty climbing onto the examination table. Coronal and axial views of a CT scan of the abdomen and
;

pelvis with contrast are shown. Which of the following active motions of the left hip is likely to be the most painful for this patient?

A ) Abduction
B ) Adduction
O G ) Extension
D ) External rotation
E ) Flexion
F ) Internal rotation

o
Previous
o
Next
(£5
Lab Values
e
Calculator
©
Review
&
Help Pause
Exam Section 4: Item SO of 50 National Board of Medical Examiners^ Time Remaining:
Mark Comprehensive Basic Science Self-Assessment 1 hr 11 min 12 sec

50. An investigator is studying the mechanisms of nerve regeneration after injury. A repetition injury is induced in a peripheral nerve in an experimental animal. 11 is found that sensory and motor axons are lost by Wallerian degeneration beyond the site of the
injury. Further study is most likely to show which of the following cell structures acting as a path - finding guide for the regeneration of the damaged axons ?

A ) Axon hillock
B ) Dendritic spine
C ) Myelin sheath
D ) Nissl substance
E ) Terminal bouton

o
Previous
o
Next
lE
Lab Values Calculator
m
Review Help Pause

You might also like